Buy BOOKS at Discounted Price

Co-ordinate Geometry

Class 10th Mathematics RD Sharma Solution
Exercise 14.1
  1. On which axis do the following points lie? (i) P (5, 0) (ii) Q (0 -2) (iii) R…
  2. Let ABCD be a square of side 2a. Find the coordinates of the vertices of this…
  3. The base PQ of two equilateral triangles PQR and PQR with side 2a lies along…
Exercise 14.2
  1. Find the distance between the following pair of points:(i) (-6,7) and (-1,…
  2. Find the value of a when the distance between the points (3, a) and (4, 1) is…
  3. If the points (2, 1) and (1,-2) are equidistant from the point (x, y), show…
  4. Find the values of x, y if the distances of the point (x, y) from (-3, 0) as…
  5. The length of a line segment is of 10 units and the coordinates of one…
  6. Show that the points A(- 4, -1), B(-2, - 4), C(4, 0) and D(2, 3) are the…
  7. Show that the points A (1,- 2), B (3, 6), C (5, 10) and D (3, 2) are the…
  8. Prove that the points A (1, 7), B (4, 2), C (-1, -1) and D (-4, 4) are the…
  9. Prove that the points (3, 0), (6, 4) and (- 1, 3) are vertices of a…
  10. Prove that (2, -2), (-2, 1) and (5, 2) are the vertices of a right angled…
  11. Prove that the points (2 a, 4 a), (2 a, 6 a) and (2a + root 3a , 5a) are the…
  12. Prove that the points (2, 3), (-4, -6) and (1, 3/2) do not form a triangle.…
  13. An equilateral triangle has two vertices at the points (3, 4) and (-2, 3),…
  14. Show that the quadrilateral whose vertices are (2, -1), (3, 4), (-2, 3) and…
  15. Two vertices of an isosceles triangle are (2, 0) and (2, 5). Find the third…
  16. Which point on x-axis is equidistant from (5, 9) and (- 4, 6)?
  17. Prove that the points (- 2, 5), (0, 1) and (2, - 3) are collinear.…
  18. The coordinates of the point P are (-3, 2). Find the coordinates of the point…
  19. Which point on y-axis is equidistant from (2, 3) and (-4, 1)?
  20. The three vertices of a parallelogram are (3, 4), (3, 8) and (9, 8). Find the…
  21. Find the circumcentre of the triangle whose vertices are (-2, -3), (- 1, 0),…
  22. Find the angle subtended at the origin by the line segment whose end points…
  23. Find the centre of the circle passing through (2, 1), (5, - 8) and (2, - 9).…
  24. Find the value of k, if the point P (0, 2) is equidistant from (3, k) and (k,…
  25. If two opposite vertices of a square are (5, 4) and (1, -6), find the…
  26. Show that the points (-3, 2), (-5, -5), (2, -3) and (4, 4) are the vertices of…
  27. Find the coordinates of the circumcentre of the triangle whose vertices are…
  28. Find a point on the x-axis which is equidistant from the points (7, 6) and…
  29. Show that the points A(5, 6), B (1, 5), C(2, 1) and D(6, 2) are the vertices…
  30. Prove that the points A (2, 3), B (-2, 2), C (-1, -2), and D (3, -1) are the…
  31. Find the point on x-axis which is equidistant from the points (-2, 5) and…
  32. Find the value of x such that pq = qr where the coordinates of P, Q and R are…
  33. Prove that the points (0, 0), (5, 5) and (-5, 5) are the vertices of a right…
  34. If the point P(x, y) is equidistant from the points A(5, 1) and B (1, 5),…
  35. Q (0, 1) is equidistant from P (5, -3) and R (x, 6), find the values of x.…
  36. Find the values of y for which the distance between the points P (2, -3) and Q…
  37. Find the centre of the circle passing through (6, -6), (3, -7) and (3, 3).…
  38. Two opposite vertices of a square are (-1, 2) and (3, 2). Find the coordinates…
  39. Name the quadrilateral formed, if any, by the following points, and give…
  40. Find the equation of the perpendicular bisector of the line segment joining…
  41. Prove that the points (3, 0), (4, 5), (-1, 4) and (-2, -1), taken in order,…
  42. In the seating arrangement of desks in a classroom three students Rohini,…
  43. Find a point on y-axis which is equidistant from the points (5, - 2) and (- 3,…
  44. Find a relation between x and y such that the point (x, y) is equidistant from…
  45. If a point A (0, 2) is equidistant from the points B (3, p) and C (p, 5), then…
  46. Prove that the points (7, 10), (-2, 5) and (3, -4) are the vertices of an…
  47. If the point P (x, 3) is equidistant from the points A (7,-1) and B (6, 8),…
  48. If A (3, y) is equidistant from points P (8, -3) and Q (7,6) , find the value…
  49. If (0, - 3) and (0, 3) are the two vertices of an equilateral triangle, find…
  50. If the point P (2, 2) is equidistant from the points A (-2, k) and B (-2k,…
  51. If the point A (0, 2) is equidistant from the points B (3, p) and C (p, 5) the…
  52. If the point P (k -1, 2) is equidistant from the points A (3, k) and B (k,5),…
Exercise 14.3
  1. Find the coordinates of the point which divides the line segment joining (- 1,…
  2. Find the points of trisection of the line segment joining the points: (i) (5,…
  3. Find the coordinates of the point where the diagonals of the parallelogram…
  4. Prove that the points (3, -2), (4, 0), (6, -3) and (5, -5) are the…
  5. Three consecutive vertices of a parallelogram are (-2, -1), (1, 0) and (4, 3).…
  6. The points (3, -4) and (-6, 2) are the extremities of a diagonal of a…
  7. Find the ratio in which the point (2,y) divides the line segment joining the…
  8. If A (-1, 3), B (1, -1) and C (5, 1) are the vertices of a triangle ABC, find…
  9. If the coordinates of the mid-points of the sides of a triangle are (1, 1), (2,…
  10. If a vertex of a triangle be (1, 1) and the middle points of the sides through…
  11. In what ratio is the line segment joining the points (-2, -3) and (3, 7)…
  12. In what ratio is the line segment joining (-3, -1) and (-8, -9) divided at…
  13. If the mid-point of the line joining (3, 4) and (k, 7) is (x, y) and 2x + 2y +…
  14. Determine the ratio in which the straight line x - y - 2 = 0 divides the line…
  15. Find the ratio in which the line segment joining (-2, -3) and (5, 6) is…
  16. Prove that the points (4, 5), (7, 6), (6, 3), (3, 2) are the vertices of a…
  17. Prove that (4, 3), (6, 4), (5, 6) and (3, 5) are the angular points of a…
  18. Prove that the points (-4, -1), (-2, -4), (4, 0) and (2, 3) are the vertices…
  19. Find the lengths of the medians of a triangle whose vertices are A (-1,3), B…
  20. Three vertices of a parallelogram are (a + b, a - b), (2a + b, 2a - b), (a -…
  21. If two vertices of a parallelogram are (3, 2), (-1, 0) and the diagonals cut…
  22. If the coordinates of the mid-points of the sides of a triangle are (3, 4),…
  23. The line segment joining the points P (3, 3) and Q (6, - 6) is trisected at…
  24. If the points (-2 , -1), (1, 0), (x, 3) and (1, y) form a parallelogram, find…
  25. The points A (2, 0), B (9, 1), C (11, 6) and D (4, 4) are the vertices of a…
  26. If three consecutive vertices of a parallelogram are (1, -2), (3, 6) and (5,…
  27. If the points A(a, -11), B(5, b), C(2, 15) and D(1, 1) are the vertices of a…
  28. If the coordinates of the mid-points of the sides of a triangle be (3, -2),…
  29. Find the lengths of the medians of a delta ABC having vertices at A (0,-1), B…
  30. Find the lengths of the medians of a delta ABC having vertices at A (5, 1), B…
  31. Find the coordinates of the points which divide the line segment joining the…
  32. Show that the mid-point of the line segment joining the points (5, 7) and (3,…
  33. Find the distance of the point (1, 2) from the mid-point of the line segment…
  34. If A and B are (1, 4) and (5, 2) respectively, find the coordinates of P when…
  35. Show that the points A (1, 0), B (5, 3), C (2, 7) and D (-2, 4) are the…
  36. Determine the ratio in which the point P (m, 6) divides the join of A(-4, 3)…
  37. Determine the ratio in which the point (-6, a) divides the join of A(-3, 1)…
  38. The line segment joining the points (3, -4) and (1, 2) is trisected at the…
  39. The line joining the points (2,1) and (5,-8) is trisected at the points P and…
  40. If A and B are two points having coordinates (-2, -2) and (2, -4)…
  41. Find the coordinates of the points which divide the line segment joining A…
  42. A (4, 2), B (6, 5) and C (1, 4) are the vertices of delta ABC. (i) The median…
  43. ABCD is a rectangle formed by joining the points A (-1, -1), B (-1, 4), C (5,…
  44. Show that A(-3, 2), B (-5, -5), C (2, -3) and D(4, 4) are the vertices of a…
  45. Find the ratio in which the y-axis divides the line segment joining the points…
  46. If the points A (6, 1), B (8, 2), C (9, 4) and D (k, p) are the vertices of a…
  47. In what ratio does the point (-4, 6) divide the line segment joining the…
  48. Find the coordinates of a point A, where AB is a diameter of the circle whose…
  49. A point P divides the line segment joining the points A (3, -5) and B (-4, 8)…
  50. Find the ratio in which the point P(-1, y) line segment joining A (-3,10) and…
  51. Points p, Q, R and S divide the segment joining the points A (1, 2) and B (6,…
  52. The mid-point P of the line segment joining the points A (- 10, 4) and B (- 2,…
  53. Find the ratio in which the point P (x, 2) divides the line segment joining…
  54. Find the ratio in which the line segment joining the points A (3,-3) and B…
  55. Find the ratio in which the points P (3/4, 5/12) divides the line segments…
  56. If the points P, Q(x, 7), R, S(6, y) in this order divide the line segment…
Exercise 14.4
  1. Find the centroid of the triangle whose vertices are: (i) (1, 4), (-1, -1), (3,…
  2. Two vertices of a triangle are (1, 2), (3, 5) and its centroid is at the…
  3. Prove analytically that the line segment joining the middle points of two sides…
  4. Prove that the lines joining the middle points of the opposite sides of a…
  5. If G be the centroid of a triangle ABC and P be any other point in the plane,…
  6. If G be the centroid of a triangle ABC, prove that: AB^2 + BC^2 + CA^2 = 3…
  7. If (-2, 3), (4, -3) and (4, 5) are the mid-points of the sides of a triangle,…
  8. In Fig. 14.40, a right triangle BOA is given. C is the mid-point of the…
  9. Find the third vertex of a triangle, if two of its vertices are at (-3, 1) and…
  10. A (3, 2) and B (-2, 1) are two vertices of a triangle ABC whose centroid G has…
Exercise 14.5
  1. Find the area of a triangle whose vertices are(i) (6, 3), (-3, 5) and (4, -…
  2. Find the area of the quadrilaterals, the coordinates of whose vertices are (i)…
  3. The four vertices of a quadrilateral are (1, 2), (-5, 6), (7, -4) and (k, -2)…
  4. The vertices of ABC are (-2, 1), (5, 4) and (2, -3) respectively. Find the area…
  5. Show that the following sets of points are collinear. (a) (2, 5), (4, 6) and…
  6. Prove that the points (a, 0), (0, b) and (1, 1) are collinear if, 1/a + 1/b = 1…
  7. The point A divides the join of P (-5, 1) and Q (3, 5) in the ratio k : 1. Find…
  8. The area of a triangle is 5. Two of its vertices are (2, 1) and (3, -2). The…
  9. If a not equal b not equal c , prove that the points (a, a^2), (b,b^2),(c, c^2)…
  10. Four points A (6, 3), B (-3, 5), C (4, - 2) and D (x, 3x) are given in such a…
  11. For what value of a the point (a, 1),(1, -1) and(11, 4) are collinear?…
  12. Prove that the points (a, b),(a1,b1) and (a - a1, b - b1) are collinear if ab1…
  13. If three points (x1, y1), (x2, y2), (x3, y3) lie on the same line, prove that…
  14. If (x, y) be on the line joining the two points (1, -3) and (-4, 2), prove…
  15. Find the value of k if points (k, 3), (6, - 2) and (-3, 4) are collinear.…
  16. Find the value of k, if the points A (7, -2), B (5, 1) and C (3, 2k) are…
  17. If the point P (m, 3) lies on the line segment joining the points A((- 2/5 ,…
  18. If R (x, y) is a point on the line segment joining the points P (a, b) and Q…
  19. Find the value of k, if the points A (8, 1), B (3, - 4) and C (2, k) are…
  20. Find the value of a for which the area of the triangle formed by the points A…
  21. If the vertices of a triangle are (1,-3), (4, p) and (-9, 7) and its area is…
  22. Find the area of a parallelogram ABCD if three of its vertices are A(2, 4), B…
  23. Find the value (s) of k for which the points (3k - 1, k - 2), (k, k - 7) and…
  24. If the points A (-1,-4), B (b,c) and C (5,-1) are collinear and 2b + c = 4,…
  25. If the points A (-2,1), B (a, b) and C (4,-1) are collinear and a - b = 1,…
  26. If A (-3, 5), B (-2,-7), C (1,-8) and D (6, 3) are the vertices of a…
  27. If P (-5, - 3), Q (-4, -6), R (2,-3) and S (1, 2) are the vertices of a…
  28. Find the area of the triangle PQR with Q (3, 2) and the mid-points of the…

Exercise 14.1
Question 1.

On which axis do the following points lie?

(i) P (5, 0) (ii) Q (0 -2)

(iii) R (- 4, 0) (iv) S (0, 5)


Answer:

P on x-axis, since ordinate is zero.

Q on y-axis, since abscissa is zero.


R on x-axis, since ordinate is zero.


S on y-axis, since abscissa is zero.



Question 2.

Let ABCD be a square of side 2a. Find the coordinates of the vertices of this square when

(i) A coincides with the origin and AB and AD are along OX and OY respectively.

(ii) The centre of the square is at the origin and coordinate axes are parallel to the sides AB and AD respectively.


Answer:

(i) Since each side of square is 2a.


Coordinates of A are (0, 0), since it coincides with origin.


Coordinates of B are (2a, 0), for a point along x-axis ordinate is zero.


Coordinates of C are (2a, 2a), since this point is equi-distance from x-axis and y-axis.


Coordinates of D are (0, 2a), since abscissa is zero and ordinate is 2a.


(ii) Each side of square is a units.


Coordinates of A are (a, a), since this point lies in Ist coordinate.


Coordinates of B are (-a, a), since this point lies in IInd coordinate.


Coordinates of C are (-a, -a), since this point lies in IIIrd coordinate.


Coordinates of D are (a, -a), since this point lies in IVth coordinate.



Question 3.

The base PQ of two equilateral triangles PQR and PQR' with side 2a lies along y-axis such that the mid-point of PQ is at the origin. Find the coordinates of the vertices R and R' of the triangles.


Answer:


Since PQ is the base of two equilateral triangles with side 2a and mid-point of PQ is at origin.


Therefore point R lies on positive x-axis and point R’ lies on negative y-axis.


OR2 = (2a)2- a2


OR2 = 4a2- a2


OR = √3a


Therefore coordinates of R are ( √3a, 0) and R’ (0, √3a)




Exercise 14.2
Question 1.

Find the distance between the following pair of points:
(i) (-6,7) and (-1, -5)
(ii) (a + b, b + c) and (a - b, c - b)
(iii) (a sin a, - b cos a) and (-a cos a, b sin a)
(iv) (a, 0) and (0, b)


Answer:

(i) (-6,7) and (-1, -5)

Distance =

Distance =

Distance =

Distance =

Distance == 13 units

Thus, the distance between the points (-6,7) and (-1, -5) is 13 units

(ii) (a + b, b + c) and (a - b, c - b)

Distance =

Distance =

Distance =

Distance =

Distance == units

Thus, the distance between these points is (a + b, b + c) and (a - b, c - b) is units.

(iii) (a sin a, - b cos a) and (-a cos a, b sin a)

Distance =

(iv) (a, 0) and (0, b)

Distance =

Distance =

Distance =
Thus, the distance between points (a, 0) and (0, b) is


Question 2.

Find the value of a when the distance between the points (3, a) and (4, 1) is .


Answer:

Distance =



= 10



a2 - 2a - 8 = 0


a2 - 4a + 2a – 8 = 0


a(a - 4) + 2(a - 4) = 0


(a - 4) (a + 2) = 0


a = 4 and a = -2



Question 3.

If the points (2, 1) and (1,-2) are equidistant from the point (x, y), show that x + 3y = 0.


Answer:

Distance from point (2, 1) = Distance from point (1, -2)



Square roots are cancelled, therefore







Question 4.

Find the values of x, y if the distances of the point (x, y) from (-3, 0) as well as from (3, 0) are 4.


Answer:

Given: the distances of the point (x, y) from (-3, 0) as well as from (3, 0) are 4.

To find: the values of x, y

Solution:

distances of the point (x, y) from (-3, 0) is

Distance =






…………….. (1)


distances of the point (x, y) from (3, 0) is


Distance =






…………….. (2)


Subtract eq 1 from eq 2 to get,

⇒ -6x + x2 + y2 - (6x + x2 + y2 ) = 7 - 7

⇒ -6x + x2 + y2 - 6x - x2 - y2 = 0

⇒ -12 x = 0

⇒ x = 0

Putting the value of x in eq 1 we get,

6x + x2 + y2 = 7

⇒ 6(0) + 02 + y2 = 7

⇒ y2 = 7

⇒ y=

Hence, X=0, y=


Question 5.

The length of a line segment is of 10 units and the coordinates of one end-point are (2,-3). If the abscissa of the other end is 10, find the ordinate of the other end.


Answer:

Let the ordinate of other end is k


Distance =



On squaring both sides, we get








; ;


Therefore ordinates are 3, -9



Question 6.

Show that the points A(- 4, -1), B(-2, - 4), C(4, 0) and D(2, 3) are the vertices points of a rectangle.


Answer:

Given: the points A(- 4, -1), B(-2, - 4), C(4, 0) and D(2, 3)

To prove: the points are the vertices points of a rectangle.

Solution:


Vertices of rectangle ABCD are: A(- 4, -1), B(-2, - 4), C(4, 0) and D(2, 3)


Length of sides=


Length of side AB = = = units


Length of side BC = = = = units


Length of side CD = = = units


Length of side AD = = = = units


Length of diagonal BD = = = units


Length of diagonal AC = = = units


Since opposite sides are equal and diagonal are equal. Therefore given vertices are the vertices of a rectangle.


Question 7.

Show that the points A (1,- 2), B (3, 6), C (5, 10) and D (3, 2) are the vertices of a parallelogram.


Answer:

Vertices of a parallelogram ABCD are: A (1,- 2), B (3, 6), C (5, 10) and D (3, 2) Length of side AB =

Length of side AB = = √(4+64)= √68 units

Length of side BC = = √(4+16) = √20 units

Length of side CD = = √(4+64)= √68 units

Length of side DA = = √(4+16) = √20 units

Length of diagonal BD = = √16= 4 units

Length of diagonal AC = = √(16+144) = √160 units

Opposite sides of the quadrilateral formed by the given four points are equal i.e. (AB = CD) & (DA = BC)
Also, the diagonals BD & AC are unequal.
Therefore, the given points form a parallelogram.


Question 8.

Prove that the points A (1, 7), B (4, 2), C (-1, -1) and D (-4, 4) are the vertices of a square.


Answer:

Vertices of a square ABCD are: A (1, 7), B (4, 2), C (-1, -1) and D (-4, 4) Length of side AB =


Length of side AB = = = units


Length of side BC = = = units


Length of side CD = = = units


Length of side DA = = = units


Length of diagonal BD = = = units


Length of diagonal AC = = = units


Since opposite sides are equal and diagonal are equal. Therefore given vertices are the vertices of a square.



Question 9.

Prove that the points (3, 0), (6, 4) and (- 1, 3) are vertices of a right-angled isosceles triangle.


Answer:

Vertices of a triangle ABC are: A(3, 0), B(6, 4) and C (- 1, 3)


Length of side AB =


Length of side AB = = = units


Length of side BC = = = units


Length of side AC = = = units


Since AB = AC, therefore triangle is an isosceles.


BC2 = AB2 + AC2


(√50)2 = (√25)2 + (√25)2


50 = 25 + 25


50 = 50


Since BC2 = AB2 + AC2; therefore given triangle is right angled triangle.



Question 10.

Prove that (2, -2), (-2, 1) and (5, 2) are the vertices of a right angled triangle. Find the area of the triangle and the length of the hypotenuse.


Answer:

Solution:

Vertices of a triangle ABC are: A(2, -2), B(-2, 1) and C(5, 2)



Length of side AB =


Length of side AB = = = units


Length of side BC = = = units


Length of side AC = = = units


Since AB = AC, therefore triangle is an isosceles.


BC2 = AB2 + AC2


(√50)2 = (√25)2 + (√25)2


50 = 25 + 25


50 = 50


Since BC2 = AB2 + AC2; therefore given triangle is right angled triangle.


Area of right angled triangle =


Area of right angled triangle = square units


Length of hypotenuse (BC) = = units


Question 11.

Prove that the points (2 a, 4 a), (2 a, 6 a) and are the vertices of an equilateral triangle.


Answer:

Vertices of a triangle ABC are: A(2 a, 4 a), B(2 a, 6 a) and C


Length of side AB =


Length of side AB = = = 2a units


Length of side BC = = a + a units


Length of side AC = = a + a units


The given vertices are not the vertices of an equilateral triangle



Question 12.

Prove that the points (2, 3), (-4, -6) and (1, 3/2) do not form a triangle.


Answer:

Let the Vertices of a triangle ABC are: A(2, 3), B(-4, -6) and C(1, 3/2)


Length of side AB =


Length of side AB = = = units


Length of side BC = = = units


Length of side AC = = = units


The given vertices do not form a triangle, since sum of two sides of a triangle are not greater than third side.



Question 13.

An equilateral triangle has two vertices at the points (3, 4) and (-2, 3), find the coordinates of the third vertex.


Answer:

Given: An equilateral triangle has two vertices at the points (3, 4) and (-2, 3)
To find: the coordinates of the third vertex.
Solution: Let the Vertices of a triangle ABC are A(3, 4) and B (-2, 3), and C(x, y),
Since it is equilateral triangle,
AB=AC=BC
Where AB,AC and BC are lengths of sides of the given triangle.
To find the length of a side use distance formula ,


Length of side AB =


Length of side AB = = = units


Length of side BC = units


Length of side AC = units
Now AB=AC


⇒(AB)2 = (BC)2


=





(AB)2 = (AC)2


=





On subtracting eqn (2) from (1), we get






...... (3)

(AC)2 = (BC)2




..... (4)

Solving equations (3) and (4), we get


; y =


Therefore coordinates of C are (, )


Question 14.

Show that the quadrilateral whose vertices are (2, -1), (3, 4), (-2, 3) and (-3, -2) is a rhombus.


Answer:

Let the Vertices of a quadrilateral are: A(2, -1), B(3, 4), C(-2, 3) and D(-3, -2)


Length of side AB =


Length of side AB = = = units


Length of side BC = = = units


Length of side CD = = = units


Length of side DA = = = units


Since all sides are of equal length, therefore it is a rhombus.



Question 15.

Two vertices of an isosceles triangle are (2, 0) and (2, 5). Find the third vertex if the length of the equal sides is 3.


Answer:

Vertices of an isosceles are: A(2, 0) and B(2, 5).


Let the third vertex is P(x, y)


Using distance formula =


Length of side PA = units


Length of side PB = units


Since PA = PB



On squaring both sides, we get




,


Also, PA = 3



On squaring both sides, we get





On substituting ,





Using quadratic formula:





Therefore coordinates of third vertex are: ;



Question 16.

Which point on x-axis is equidistant from (5, 9) and (- 4, 6)?


Answer:

Since the point is on x-axis, therefore coordinate of y-axis is zero.


Therefore the point is P(k, 0) which is equidistance from A(5, 9) and B(- 4, 6)


PA = PB



On squaring both sides







Therefore coordinate is (27, 0)



Question 17.

Prove that the points (- 2, 5), (0, 1) and (2, - 3) are collinear.


Answer:

Vertices are: A(- 2, 5), B(0, 1) and C(2, - 3)


Using distance formula =


Length of side AB = = = = units


Length of side BC = = = = units


Length of side AC = = = = units


Since length of AB + BC = AC, therefore points are collinear.



Question 18.

The coordinates of the point P are (-3, 2). Find the coordinates of the point Q which lies on the line joining P and origin such that OP = OQ.


Answer:

Let the coordinates of Point Q are (x, y) and coordinates of origin O are (0, 0)


Since OP = OQ


Points are: P (-3, 2), Q(x, y) and O (0, 0)


Q is the mid point






Therefore coordinates are (3, -2)



Question 19.

Which point on y-axis is equidistant from (2, 3) and (-4, 1)?


Answer:

Since the point is on y-axis, therefore coordinate of x-axis is zero.


Therefore the point is P(0, k) which is equidistance from A(2, 3) and B(- 4, 1)


PA = PB




On squaring both sides, we get




Therefore coordinate is (0, -1)



Question 20.

The three vertices of a parallelogram are (3, 4), (3, 8) and (9, 8). Find the fourth vertex.


Answer:

Consider A(3, 4), B (3, 8) and C(9, 8).


Let the coordinates of fourth vertex are D (x, y)


In a parallelogram diagonals bisect each other


Coordinate of mid point of AC =



Therefore coordinates of mid point of AC are (6, 6)


Coordinate of mid point of BD =



Coordinates of point D are






Therefore coordinates of fourth vertex D are (9, 4)



Question 21.

Find the circumcentre of the triangle whose vertices are (-2, -3), (- 1, 0), (7, - 6).


Answer:

Vertices of triangle are A(-2, -3), B(- 1, 0), C(7, - 6)


Let the coordinates of P are (x, y)


PA = PB =PC


PA = PB



On squaring both sides, we get




…………(1)


PA = PC



On squaring both sides, we get




…………(2)


On solving equations (1) & (2), We get


X = 3 and y = -3


Therefore coordinates are (3, -3)



Question 22.

Find the angle subtended at the origin by the line segment whose end points are (0,100) and (10, 0).


Answer:

Since the abscissa of first coordinate is zero, therefore this point lies on y-axis. Ordinate of second point is zero, therefore this point lies on y-axis. We know that both the axes are perpendicular to eachother, therefore the angle between these points is 90°.



Question 23.

Find the centre of the circle passing through (2, 1), (5, - 8) and (2, - 9).


Answer:

Coordinates of points on a circle are A(2,1), B(5,-8) and C(2,-9).


Let the coordinates of the centre of the circle be O(x, y)


Using distance formula =


Since the distance of the points A, B and C will be equal from the center, therefore


⇒ OA = OB



On squaring both sides, we get


⇒ x2 + 4 - 4x + y2 + 1 - 2y = x2 + 25 - 10x + y2 + 64 + 16y


⇒ 6x - 18y - 84 = 0


⇒ x - 3y - 14 = 0 ------------- (1)


Similarly, OC = OB


⇒ x2 + 4 - 4x + y2 + 81 + 18y = x2 + 25 - 10x + y2 + 64 + 16y


⇒ 6x - 2y - 4 = 0


⇒ 3x - y - 2 = 0 ------------- (2)


By solving equations (1) and (2), we get x = -1, y = -5


So, the coordinates of the centre of the circle is (-1, -5).


Radius of the circle = OA =


=


=


= units



Question 24.

Find the value of k, if the point P (0, 2) is equidistant from (3, k) and (k, 5).


Answer:

Let the point is P(0, 2) which is equidistance from A(3, k) and B(k, 5)


PA = PB



On squaring both sides, we get






Question 25.

If two opposite vertices of a square are (5, 4) and (1, -6), find the coordinates of its remaining two vertices.


Answer:

Let ABCD is a square with A(5, 4) and C(1, -6).


Let the coordinates of B are (x, y)


Using distance formula =


⇒ AB = BC



On squaring both sides, we get





……………………….(1)


In ΔABC, Using Pythagoras theorem


AC2 = AB2 + BC2


AC2 = 2BC2 [Since AB = BC]



On squaring both sides, we get





………………..(2)


On substituting value of from equation (1) in equation (2), we get







On solving we get y = -3, -1


Substituting these values of y in eqn 1, we get x = 8, -2


Therefore other coordinates are B(8, -3). And D(-2,1)



Question 26.

Show that the points (-3, 2), (-5, -5), (2, -3) and (4, 4) are the vertices of a rhombus. Find the area of this rhombus.


Answer:

Vertices of the rhombus are: A(-3, 2), B(-5, -5), C(2, -3) and D(4, 4)


We know that diagonals of a rhombus bisect each other, therefore point of intersection of diagonals is:


Abscissa of Mid point of AC =


Ordinate of Mid point of AC =


Abscissa of Mid point of BD =


Ordinate of Mid point of BD =


Since the diagonals AC and BD bisect each other at O, therefore it is a rhombus.


Length of diagonal AC = = = = units


Length of diagonal BD = = = = units


Area of rhombus = = = 45 sq units


Area of rhombus is 45 sq units



Question 27.

Find the coordinates of the circumcentre of the triangle whose vertices are (3, 0), (-1, -6) and (4,-1). Also, find its circumradius.


Answer:

Coordinates of points on a circle are A (3, 0), B(-1, -6) and C(4,-1)


Let the coordinates of the centre of the circle be O(x, y)


Using distance formula =


Since the distance of the points A, B and C will be equal from the center, therefore


⇒ OA = OC



On squaring both sides, we get


=


⇒ x2 + 9 - 6x + y2 = x2 + 2x +1 + y2 + 36 + 12y


⇒ -8x - 12y = 28


⇒ 2x + 3y = -7------------- (1)


Similarly, OC = OB



On squaring both sides, we get


=


⇒ x2 + 16 - 8x + y2 + 1 + 2y = x2 + 1 +2x + y2 + 36 + 12y


⇒ -10x - 10y = 20


⇒ x + y = -2 ------------- (2)


Solving eqn (1) and (2), we get


x = 1; y = -3


Coordinates of circum center are (1, -3)


Circum radius of the circle = OA =


=


= units



Question 28.

Find a point on the x-axis which is equidistant from the points (7, 6) and (-3, 4).


Answer:

points A(7, 6) and B(-3, 4) are equidistance from point P.


Let the coordinates of point are P(x, 0)


Using distance formula =


⇒ PA = PB



On squaring both sides, we get


=





Therefore coordinates are (3, 0)



Question 29.

Show that the points A(5, 6), B (1, 5), C(2, 1) and D(6, 2) are the vertices of a square.


Answer:

Vertices of a quadrilateral are A(5,6), B(1,5), C(2,1) and D(6,2).


Using distance formula =


AB = = = units


BC = = = units


CD = = = units


DA = = = units


AB = BC = CD = DA


BD = = = units


AC = = = units


All the four sides of the quadrilateral are equal and diagonals are of equal length. Therefore, the given vertices form a square.



Question 30.

Prove that the points A (2, 3), B (-2, 2), C (-1, -2), and D (3, -1) are the vertices of a square ABCD.


Answer:

The Vertices of a quadrilateral are A(5,6), B(1,5), C(2,1) and D(6,2).

Using distance formula =

AB = = = units

BC = = = units

CD = = = units

DA = = = units

AB = BC = CD = DA

BD = = = units

AC = = = units

All the four sides of the quadrilateral are equal and diagonals are of equal length. Therefore, the given vertices form a square.


Question 31.

Find the point on x-axis which is equidistant from the points (-2, 5) and (2,-3).


Answer:

Points A(-2, 5) and B(2, -3) are equidistant from point P.

Let the coordinates of point are P(x, 0)

Using distance formula =

⇒ PA = PB

On squaring both sides, we get

=

8x = -16
x = -2

Hence, coordinates are (-2, 0).


Question 32.

Find the value of x such that where the coordinates of P, Q and R are (6,-1), (1, 3) and (x, 8) respectively.


Answer:

Coordinates are P(6,-1), Q(1, 3) and R(x, 8)


Using distance formula =


⇒ PQ = QR



On squaring both sides, we get


=


=



On solving above equation, we get







Therefore x = -3, 5



Question 33.

Prove that the points (0, 0), (5, 5) and (-5, 5) are the vertices of a right isosceles triangle.


Answer:

Vertices of a quadrilateral are A (0, 0), B(5, 5) and C(-5, 5)


Using distance formula =


AB = = = units


BC = = = units


CA = = = units


Since AB = CA


Using Pythagoras theorem


BC2 = AC2 + AB2


100 = 50 + 50


100 = 100


Therefore vertices are of right isosceles triangle.



Question 34.

If the point P(x, y) is equidistant from the points A(5, 1) and B (1, 5), prove that x = y.


Answer:

Coordinates are P(x, y), A(5, 1) and B (1, 5)


Using distance formula =


⇒ PA = PB



On squaring both sides, we get


=


=


=


proved



Question 35.

Q (0, 1) is equidistant from P (5, -3) and R (x, 6), find the values of x. Also, find the distances QR and PR.


Answer:

Coordinates are Q(0, 1), P (5, -3) and R (x, 6),


Using distance formula =


⇒ QP = QR



On squaring both sides, we get


=


=


=


proved


QR = = = units


PR = = = units



Question 36.

Find the values of y for which the distance between the points P (2, -3) and Q (10, y) is 10 units.


Answer:

Given: the distance between the points P (2, -3) and Q (10, y) is 10 units.
To find: The value of y.
Solution:
Coordinates are P (2, -3) and Q (10, y)


We use distance formula to find the distance between two points.


Since PQ = 10 units
S0,

= 10


On squaring both sides, we get


= 100


⇒ 82 + (y+3)2 = 100

= 100
⇒ 73 + y2 + 6y = 100
⇒ 73 + y2 + 6y -100 = 0
= 0
In factorization, we write the middle term of the quadratic equation either as a sum of two numbers or difference of two numbers such that the equation can be factorized.

⇒ y2 + 9y - 3y - 27 = 0

⇒y(y + 9) - 3(y + 9) = 0

⇒(y - 3)(y + 9) = 0

⇒y = 3, -9


Question 37.

Find the centre of the circle passing through (6, -6), (3, -7) and (3, 3).


Answer:

Coordinates of points on a circle are A(6, -6), B(3, -7) and C(3, 3).


Let the coordinates of the centre of the circle be O(x, y)


Using distance formula =


Since the distance of the points A, B and C will be equal from the center, therefore


⇒ OA = OC



On squaring both sides, we get


=


=


= ………………….(1)


Similarly, OA = OB



On squaring both sides, we get


=


=


= …………..(1)


Solving eqn (1) and (2), we get


x = 3; y = -2


Coordinates of circum center are (3, -2)



Question 38.

Two opposite vertices of a square are (-1, 2) and (3, 2). Find the coordinates of other two vertices.


Answer:

The coordinates are A(-1, 2) and C(3, 2).


Let the coordinates of the vertex B are (x, y)


AB = BC


Using distance formula =



On squaring both sides, we get


=


=


=


In ΔABC


AB2 + BC2 = AC2 [Using Pythagoras theorem]


2AB2 = AC2 [Since AB = BC]


=


= 16


= 8


= 3


On substituting x = 1


= 3


= 0


= 0


= 0, 4


Other coordinates are (1, 0) and (1, 4)



Question 39.

Name the quadrilateral formed, if any, by the following points, and give reasons for your answers:

(i) A (-1, - 2), B (1, 0), C (-1, 2), D (-3, 0)

(ii) A (-3, 5), B (3, 1), C (0, 3), D (-1, - 4)

(iii) A (4, 5), B (7, 6), C (4, 3), D (1, 2)


Answer:

(i) A (-1, - 2), B (1, 0), C (-1, 2), D (-3, 0)


Using distance formula:


AB = BC = CD = DA




All the four sides of the quadrilateral are equal and diagonals are of equal length. Therefore, the given vertices form a square.


(ii) A (-3, 5), B (3, 1), C (0, 3), D (-1, - 4)


Using distance formula:





Since all sides are of different length, therefore it is not a particular type of quadrilateral.


(iii) A (4, 5), B (7, 6), C (4, 3), D (1, 2)



Using distance formula





Coordinates of midpoint of diagonal AC
X=
Y =


Therefore coordinates of midpoint of AC are (4, 4)


Coordinates of midpoint of diagonal BD=
X = , Y =


Therefore coordinates of midpoint of AC are (4, 4)


Since diagonals bisect each other at same point therefore quadrilateral is a parallelogram.


Question 40.

Find the equation of the perpendicular bisector of the line segment joining points (7, 1) and (3, 5).


Answer:

The points are A(7, 1) and B(3, 5).


Coordinates of midpoint of line AB = X = , Y =


Therefore coordinates of midpoint of AB are (5, 3)


Slope of the line = = = = 2


Negative reciprocal of slope =


Equation of line Y= mX+C








Since diagonals bisect each other at same point therefore quadrilateral is a parallelogram.



Question 41.

Prove that the points (3, 0), (4, 5), (-1, 4) and (-2, -1), taken in order, form a rhombus. Also, find its area.


Answer:

Let the Vertices of a quadrilateral are: A(3, 0), B(4, 5), C(-1, 4) and D(-2, -1),


Length of side AB =


Length of side AB = = = units


Length of side BC = = = units


Length of side CD = = = units


Length of side DA = = = units


Length of diagonal AC = = = units


Length of side BD = = = units


Since all sides are of equal length, therefore it is a rhombus.


Area of Rhombus = = = = 24 sq. units



Question 42.

In the seating arrangement of desks in a classroom three students Rohini, Sandhya and Bina are seated at A (3, 1), B (6, 4) and C (8, 6). Do you think they are seated in a line?


Answer:

Points are A (3, 1), B (6, 4) and C (8, 6)


For sitting in a line three points must be collinear i.e AB + BC = AC


Length of side AB =


Length of side AB = = = = units


Length of side BC = = = = units


Length of side AC = = = units


AB + BC = AC


+ =


The points are collinear.



Question 43.

Find a point on y-axis which is equidistant from the points (5, - 2) and (- 3, 2).


Answer:

Points A(5, -2) and B(-3, 2) are equidistance from point P.


Let the coordinates of point are P(0, y)


Using distance formula =


⇒ PA = PB



On squaring both sides, we get

(0-5)2 + (y+2)2 = (0+3)2 + (y-2)2

⇒ (-5)2 + (y+2)2 = (3)2 + (y-2)2

⇒ 25 + y2 + 4 + 4y = 9 + y2 + 4 – 4y

⇒ 4y + 4y = 9 – 25

⇒ 8y = -16

⇒ y = -2

Therefore coordinates are (0, -2).


Question 44.

Find a relation between x and y such that the point (x, y) is equidistant from the points (3, 6) and (-3, 4).


Answer:

Coordinates of the points are A(3, 6) and B(-3, 4)


Let the point P(x, y) is equidistant from A and B


Using distance formula =


⇒ PA = PB



On squaring both sides, we get


=





Question 45.

If a point A (0, 2) is equidistant from the points B (3, p) and C (p, 5), then find the value of p.


Answer:

Using distance formula =


⇒ AB = AC



On squaring both sides, we get


=





Question 46.

Prove that the points (7, 10), (-2, 5) and (3, -4) are the vertices of an isosceles right triangle.


Answer:

Vertices of a quadrilateral are A (7, 10), B(-2, 5) and C(3, -4)


Using distance formula =


AB = = = units


BC = = = units


AC = = = units


Since AB = BC


Using Pythagoras theorem


AC2 = AB2 + BC2



212 = 106 + 106


212 = 212


Therefore vertices are of right isosceles triangle.



Question 47.

If the point P (x, 3) is equidistant from the points A (7,-1) and B (6, 8), find the value of x and find the distance AP.


Answer:

Coordinates are A (7,-1) and B (6, 8)


The point P (x, 3) is equidistant.


Using distance formula =


⇒ PA = PB



On squaring both sides, we get


=


=


=


AP = = = units



Question 48.

If A (3, y) is equidistant from points P (8, -3) and Q (7,6) , find the value of y and find the distance AQ.


Answer:

Coordinates are P(8, -3) and Q(7,6)


The point A (3, y) is equidistant.


Using distance formula =


⇒ PA = QA



On squaring both sides, we get


=


=


= 1


AQ = = = units



Question 49.

If (0, - 3) and (0, 3) are the two vertices of an equilateral triangle, find the coordinates of its third vertex.


Answer:

Coordinates are A(0, -3) and B(0,3) and C(x, y)


Using distance formula =


⇒ AB = AC



On squaring both sides, we get


=


=


(1)


⇒ AB = BC



On squaring both sides, we get


=


=


(2)


On subtracting equation (2) from (1) we get




On substituting y = 0 in equation (1), we get





Therefore coordinates of third vertex are (), (),



Question 50.

If the point P (2, 2) is equidistant from the points A (-2, k) and B (-2k, -3), find k. Also, find the length of AP.


Answer:

Coordinates of points are P(2, 2) A(-2, k) and B(-2k, -3)


Using distance formula =


⇒ PA = PB



On squaring both sides, we get


=


=


0


0


0


0


-1, -3


AP = = = 5 units



Question 51.

If the point A (0, 2) is equidistant from the points B (3, p) and C (p, 5) the length of AB.


Answer:

Coordinates of points are A(0, 2), B(3, p) and C(p, 5)

Using distance formula =

⇒ AB = AC

On squaring both sides, we get

=

=

1

AB = = = units


Question 52.

If the point P (k -1, 2) is equidistant from the points A (3, k) and B (k,5), find the value of k.


Answer:

Coordinates of points are A(3, k), B(k, 5) and P(k-1, 2)


Using distance formula =


⇒ PA = PB



On squaring both sides, we get


=


=


= 1 + 9


= 0


= 0


= 0


= 0


= 1, 5




Exercise 14.3
Question 1.

Find the coordinates of the point which divides the line segment joining (- 1, 3) and (4, -7) internally in the ratio 3 : 4.


Answer:

Let our points be A(-1, 3) and B(4, -7) and required point be C( x, y)



Given that point divides internally in ratio of 3:4.


By section formula,


x = , y =


Here, m = 3 and n = 4


∴ x = , y =


∴ x= , y =


∴ x = , y =


Hence, the required point is C( , )



Question 2.

Find the points of trisection of the line segment joining the points:

(i) (5, -6) and (- 7, 5), (ii) (3, -2) and (-3, -4), (iii) (2, -2) and (-7, 4)


Answer:

(i) (5, -6) and (- 7, 5),


Let our given points be A(5,-6) and B(-7, 5) and required points be C (x1 , y1 ) and D(x2 , y2)


The points of trisection of a line are points which divide into the ratio 1:2



By section formula,


x = , y =


For point C(x1 , y1 )


x1 = , y1= …Here m = 1 and n = 2


∴ x1 = , y1 =


∴ C (x1 , y1 ) ≡ (1, )


For point D(x2 , y2 )


X2 = , y2= …Here m = 2 and n = 1


∴ x2 = , y2 =


∴ D (x2 , y2)≡ (-3, )


Hence, the points of trisection of line joining given points are (1, ) and (-3, )


(ii) (3, -2) and (-3, -4)


Let our given points be A(3,-2) and B(-3, -4) and required points be C (x1 , y1 ) and D(x2 , y2)


The points of trisection of a line are points which divide into the ratio 1:2



By section formula,


x = , y =


For point C(x1 , y1 )


x1 = , y1= …Here m = 1 and n = 2


∴ x1 = , y1 =


∴ C (x1 , y1 ) ≡ ( , )


For point D(x2 , y2 )


X2 = , y2= …Here m = 2 and n = 1


∴ x2 = , y2 =


∴ D (x2 , y2)≡ (-1, )


Hence, the points of trisection of line joining given points are ( , ) and (-1, )


(iii) (2, -2) and (-7, 4)


Let our given points be A(2,-2) and B(-7, 4) and required points be C (x1 , y1 ) and D(x2 , y2)


The points of trisection of a line are points which divide into the ratio 1:2



By section formula,


x = , y =


For point C(x1 , y1 )


x1 = , y1= …Here m = 1 and n = 2


∴ x1 = , y1 =


∴ C (x1 , y1 ) ≡ ( , )


For point D(x2 , y2 )


X2 = , y2= …Here m = 2 and n = 1


∴ x2 = , y2 =


∴ D (x2 , y2)≡ (-4, 2)



Question 3.

Find the coordinates of the point where the diagonals of the parallelogram formed by joining the points (-2, -1), (1, 0), (4, 3) and (1, 2) meet.


Answer:

Let our points of parallelogram be A(-2, -1), B(1, 0), C(4, 3) and D(1, 2) and mid point of diagonals be E(x,y)



We know that diagonals of parallelogram bisect each other.


Hence, we find mid point of AC.


By midpoint formula,


x = , y =


For point E(x, y)


x1 = , y1=


∴ x1 = , y1 =


∴ E (x, y) ≡ ( , 1 )



Question 4.

Prove that the points (3, -2), (4, 0), (6, -3) and (5, -5) are the vertices of a parallelogram.


Answer:

We know if the quadrilateral is parallelogram if opposite sides are equal.


Let our points be A(3, -2), B(4, 0), C(6, -3) and D(5, -5).



By distance formula,


XY =


For AB,


AB =


=


= units


For BC,


BC =


=


= units


For CD,


CD =


=


= units


For AD,


AD =


=


= units


Here, we observe that AB = CD and AD = BC, which means that the quadrilateral formed by lines joining by points, is parallelogram.



Question 5.

Three consecutive vertices of a parallelogram are (-2, -1), (1, 0) and (4, 3). Find the fourth vertex


Answer:

Let three vertices be A(-2, -1), B(1, 0) and C(4, 3) and fourth vertex be D(x, y)



It is given that quadrilateral joining these four vertices is parallelogram.


∴□ABCD is parallelogram


We know that diagonals of parallelogram bisect each other, ie midpoint of the diagonals coincide.


Let E(xm , ym) be the midpoint of diagonals AC and BD.


By midpoint formula,


x = , y =


For diagonal AC,


xm = , ym =


∴ xm = , ym =


∴ E(xm , ym) ≡ (1, 1)


For diagonal BD,


1 = , 1 =


∴ x = 2 – 1 , y = 2 – 0


∴ x = 1 and y = 2


Hence, our fourth vertex is D(1, 2)



Question 6.

The points (3, -4) and (-6, 2) are the extremities of a diagonal of a parallelogram. If the third vertex is (-1, -3). Find the coordinates of the fourth vertex.


Answer:

Let three vertices be A(3, -4), B(-1, -3) and C(-6, 2) and fourth vertex be D(x, y)



It is given that quadrilateral joining these four vertices is parallelogram.


∴□ABCD is parallelogram


We know that diagonals of parallelogram bisect each other, ie midpoint of the diagonals coincide.


Let E(xm , ym) be the midpoint of diagonals AC and BD.


By midpoint formula,


x = , y =


For diagonal AC,


xm = , ym =


∴ xm = , ym =


∴ E(xm , ym) ≡ (, -1)


For diagonal BD,


= , -1 =


∴ x = -3 + 1 , y = -2 + 3


∴ x = -2 and y = 1


Hence, our fourth vertex is D(-2, 1)



Question 7.

Find the ratio in which the point (2,y) divides the line segment joining the points A (-2, 2) and B ( 3, 7). Also, find the value of y.


Answer:

Here, given points are A (-2, 2) and B ( 3, 7) and let the point dividing the line joining two points be C(2,y).



Let the ratio be m:n


By section formula,


x = , y =


For point C(2,y),


2 = …(1)


And y = …(2)


Solving 1 for finding ratio between m and n,


2 =


2(m + n) = 3m -2n


2m + 2n = 3m – 2n


∴ m = 4n


=


∴ m : n = 4 : 1


Now solving for equation 2, where m = 4 and n = 1


y =


y =


∴ y =


∴ y =


∴ y =6


Hence, our point is (2, 6)



Question 8.

If A (-1, 3), B (1, -1) and C (5, 1) are the vertices of a triangle ABC, find the length of the median through A.


Answer:

Here given vertices of triangle are A (-1, 3), B (1, -1) and C (5, 1).


Let D, E and F be the midpoints of the sides BC, CA and AB respectively.



We need to find length of median passing through A, ie distance between AD.


Let point D ≡ (x, y)


By midpoint formula,


x = , y =


For midpoint D of side BC,


x = , y =


∴ x = , y =


∴D(x , y) ≡ (3, 0 )


Now, by distance formula,


XY =


For AD,


AD =


∴ AD =


AD =


∴AD = 5 units


Hence, the length of the median through A is 5 units



Question 9.

If the coordinates of the mid-points of the sides of a triangle are (1, 1), (2, -3) and (3, 4), find the vertices of the triangle.


Answer:

Let A(x1 , y1), B(x2 , y2) and C(x3 , y3) be the vertices of triangle.



Let D(1, 1), E(2, -3) and F(3, 4) be the midpoints of sides BC, CA and AB respectively.


By midpoint formula.


x = , y =


For midpoint D(1, 1) of side BC,


1 = , 1 =


= 2 and = 2 …(1)


For midpoint E(2, -3) of side CA,


2 = , -3 =


= 4 and = -6 …(2)


For midpoint F(3, 4) of side AB,


3 = , 4 =


= 6 and = 8 …(3)


Adding 1,2 and 3, we get,


= 2 + 4 + 6


And = 2 – 6 +8


∴ 2(= 12 and 2() = 2


= 6 and = 1


+ 2 = 6 and + 2 = 2 …from 1


= 4 and = 0


Substituting above values in 3,


4 + and 0 + = 8


= 2 and = 89


Similarly for equation 2,


4 + = 6 and 0 + = -6


= 2 and = -6


Hence the vertices of triangle are A(4 , 0), B(2 ,8) and C(0 ,-6)



Question 10.

If a vertex of a triangle be (1, 1) and the middle points of the sides through it be (-2, 3) and (5, 2), find the other vertices.


Answer:

Let in ∆ABC, A(1,1), B(x1, y1) and C(x2, y2).


Let D(-2, 3) and E(5, 2) be the midpoints of sides AB and AC respectively.



By midpoint formula.


x = , y =


For mid point D (-2, 3) of side AB,


-2 = , 3 =


1 + x1 = -4 and 1 + y1 = 6


x1 = -5 and y1 = 5


∴ B ( x1, y1) ≡ (-5, 5)


For midpoint E(5, 2) of side AC,


5 = , 2 =


1 + x2 = 10 and = 4


∴ x2 = 9 and = 3


∴ C(x2, y2) ≡ (9, 3)


Hence other two vertices are B (-5, 5) and C(9, 3)



Question 11.

In what ratio is the line segment joining the points (-2, -3) and (3, 7) divided by the y-axis? Also, find the coordinates of the point of division.


Answer:

Here y axis divides our line joined by the points (say) A (-2, -3) and B (3, 7).


Let coordinate of the point be C(0, y).


Here our x- coordinate is zero, as point C lie on x-axis.


Let y axis divide AB in ratio of m:n.



By section formula,


x = , y =


For point C(0, y) on line joined by the points A and B,


0 = …(1)


And, y = …(2)


Solving 1,


0(3m – 2n


∴ 3m = 2n


∴m:n = 2:3


Now solving for 2, for values m = 2 and n = 3,


y =


y =


∴ y = = 1


∴ C(0, y) ≡ (0, 1)



Question 12.

In what ratio is the line segment joining (-3, -1) and (-8, -9) divided at the point (-5, -21 /5)?


Answer:

Let given points be A (-3, -1) and B (-8, -9).


Let the point C(-5, -21 /5) divide AB in ratio m:n.



By section formula,


x = , y =


For point C(-5, -21 /5) on the line joined by the points A and B.


-5 = …(1)


And, - = …(2)


Solving 1,


-5(m + n) = -8m - 3n


∴5m + 5n = 8m + 3n


∴ 2n = 3m


=


Hence, ratio is 2:3.



Question 13.

If the mid-point of the line joining (3, 4) and (k, 7) is (x, y) and 2x + 2y + 1 = 0, find the value of k.


Answer:

Let A(3, 4) and B(k, 7) and midpoint be C(x, y) which lies on the line 2x+2y+1 = 0

By midpoint formula,

x = , y =

For point C(x, y),

x = , y = …(1)

Here, y = ,

Hence, substituting value of y in given equation of line,

2x + 2 × + 1 = 0

∴ 2x = -12

∴ x = -6

Now substituting value of x in equation(1), we get.

x =

-6 =

∴ -12 = 3 + k

∴ k = -15

Hence, the value of k is -15.


Question 14.

Determine the ratio in which the straight line x - y - 2 = 0 divides the line segment joining (3, -1) and (8, 9).


Answer:

Let point be A(3, -1) and B(8, 9).


Let the line divide the line joining the points A and B in the ratio m:n at any point C(x, y)



By section formula,


x = , y =


For point C(x, y),


x = , y =


∴ x = , y =


Now, substituting value of x and y in equation x - y - 2 = 0,


- -2 = 0


= 0


∴ -3m +2n =0


=


∴ m:n = 2:3


Hence, the line divides the line segment joining A and B in the ratio 2:3 internally.



Question 15.

Find the ratio in which the line segment joining (-2, -3) and (5, 6) is divided by (i) x-axis (ii) y-axis. Also, find the coordinates of the point of division in each case.

(i) x-axis


Answer:

(i) x-axis


Let our points be A(-2, -3) and B(5, 6).


Let point C(x, 0) divide the line formed by joining by the points A and B in ratio of m:n.



By section formula,


x = , y =


For point C(x, 0)


x = , 0 =


Solving for y coordinate,


0 =


∴ 6m -3n = 0


∴ 2m = n


=


∴ m : n = 1 : 2


Now solving for x coordinate, with m = 1 and n = 2,


x =


∴ x=


∴ x =


Hence, the coordinates of required point is C( , 0)


(ii) y-axis.


Let our points be A(-2, -3) and B(5, 6).


Let point C(0, y) divide the line formed by joining by the points A and B in ratio of m:n.



By section formula,


x = , y =


For point C(0, y)


0 = , y =


Solving for x coordinate,


0 =


∴ 5m – 2n = 0


=


∴ m : n = 2 : 5


Now solving for y coordinate, with m = 2 and n = 5,


y =


y =


∴ y =


Hence, the coordinates of required point is C( , )



Question 16.

Prove that the points (4, 5), (7, 6), (6, 3), (3, 2) are the vertices of a parallelogram. Is it a rectangle.


Answer:

Let given points be A(4, 5), B(7, 6), C(6, 3), D(3, 2) and let the intersection of diagonals be E(xm , ym )



By midpoint formula.


x = , y =


For midpoint of diagonal AC,


X1 = , y1 =


∴x1 = = 5 , y1 = = 4


∴ midpoint of diagonal AC is (x1, y1 ) ≡ (5, 4) …(1)


For midpoint of diagonal BD,


X2 = , y2 =


∴x2 = = 5 , y 2= = 4


∴ midpoint of diagonal BD is (x2, y2 ) ≡ (5, 4) …(2)


Here, from 1 and 2 we say that midpoint of both the diagonals intersect at same point, ie (5, 4)


But our intersection of diagonals is at E, which means that midpoint of diagonals intersect at single point, ie E(5, 4)


We know that if midpoints of diagonals intersect at single point, then quadrilateral formed by joining the points is parallelogram.


Hence, our □ABCD is parallelogram.


Now, we shall check whether □ABCD is rectangle.


If the lengths of diagonals are same, then given quadrilateral is rectangle.


By distance formula,


XY =


For diagonal AC,


AC =


=


= units


For diagonal BD,


AC =


=


= units.


Here, AC ≠ BD, hence □ABCD is not rectangle.



Question 17.

Prove that (4, 3), (6, 4), (5, 6) and (3, 5) are the angular points of a square.


Answer:

Let given points be A(4, 3), B(6, 4), C(5, 6) and D(3, 5).



By distance formula,


XY =


For AB,


AB =


=


= units.


For BC,


BC =


=


= units.


For CD,


CD =


=


= units.


For AD,


AD =


=


= units.


Here, we can observe that □ABCD is a parallelogram.


Now,


For diagonal AC,


AC =


=


= units.


For diagonal BD,


BD =


=


= units.


∴ AC = BD, which means diagonals are equal.


We know that quadrilateral in which all sides are equal and diagonals are equal, is a square.


∴□ABCD is a square.



Question 18.

Prove that the points (-4, -1), (-2, -4), (4, 0) and (2, 3) are the vertices of a rectangle.


Answer:

Solution : Let the given points be A(-4, -1), B(-2, -4), C(4, 0) and D(2, 3).



Use distance formula,


For AB,


AB =





=


=


= units


For BC,


BC =







=


= units


For CD,






=


= units


For AD,


AD =







=


= units


Also, for diagonal AC,


AC =









= units


For diagonal BD,


BD =








=


= units


We can observe that AB = CD and BC = AD and also diagonal AC = BD.


We know that a quadrilateral whose opposite sides are equal and the diagonal are equal is rectangle.


∴ ABCD is a rectangle.


Question 19.

Find the lengths of the medians of a triangle whose vertices are A (-1,3), B (1,-1) and C(5,1).


Answer:

Here given vertices are A(-1,3), B (1,-1) and C(5,1) and let midpoints of BC, CA and AB be D,E and F respectively.



By midpoint formula.


x = , y =


For midpoint D of side BC,


x = , y =


x = , y =


∴midpoint of side BC is D(3, 0)


For midpoint E of side AB,


x = , y =


x = , y =


∴midpoint of side AB is E(2, 2)


For midpoint F of side CA,


x = , y =


x = , y =


∴midpoint of side CA is F(0, 1)


By distance formula,


XY =


For median AD,


AD =


=


=


= 5 units


For median BE,


BE =


=


= units.


For median CF,


CF =


=


= 5 units



Question 20.

Three vertices of a parallelogram are (a + b, a - b), (2a + b, 2a - b), (a - b, a + b). Find the fourth vertex.


Answer:

Let A(a + b, a - b), B(2a + b, 2a - b), C(a - b, a + b) and fourth vertex be D(x, y).



It is given that □ABCD is parallelogram.


We know that diagonals of parallelogram bisect each other.


Let intersection of diagonals be E(xm, ym )


By midpoint formula.


xm = , ym =


For midpoint E of diagonal AC,


xm = , ym =


∴ xm = a , ym = a


∴E(xm, ym ) ≡ (a, a)


For diagonal BD,


a = , a=


∴ 2a = 2a + b +x , 2a = 2a – b +y


∴ x = -b and y = b


Hence, the fourth vertex is D(-b, b)



Question 21.

If two vertices of a parallelogram are (3, 2), (-1, 0) and the diagonals cut at (2, -5), find the other vertices of the parallelogram.


Answer:

Let the vertices be A(3, 2), B(-1, 0), C(x1, y1 ) and D(x2, y2 ).


Let diagonals cut at E(2, - 5).



We know that mid points of diagonals of parallelogram coincide.


By midpoint formula.


xm = , ym =


For midpoint E of diagonal AC,


2 = , -5 =


∴x1 = 1 and y1 = -12


∴ coordinates of C are (1, -12)


For midpoint E of diagonal BD,


2 = , -5 =


∴ x2 = 5 and y2 = -10


∴ coordinates of D are (5, -10)



Question 22.

If the coordinates of the mid-points of the sides of a triangle are (3, 4), (4, 6) and (5, 7), find its vertices.


Answer:

Let A(x1 , y1), B(x2 , y2) and C(x3 , y3) be the vertices of triangle.


Let D(3, 4), E(4, 6) and F(5, 7) be the midpoints of sides BC, CA and AB respectively.



By midpoint formula.


x = , y =


For midpoint D(3, 4) of side BC,


3 = , 4 =


= 6 and = 8 …(1)


For midpoint E(4, 6) of side CA,


4 = , 6 =


= 8 and = 12 …(2)


For midpoint F(5, 7) of side AB,


5 = , 7 =


= 10 and = 14 …(3)


Adding 1,2 and 3, we get,


=6 + 8 + 10


And = 8 + 12 + 14


∴ 2(= 24 and 2() = 34


= 12 and = 17


+ 6 = 12 and + 8 = 17 …from 1


= 6 and = 9


Substituting above values in 3,


6 + and 9 + = 14


= 4 and = 5


Similarly for equation 2,


6 + = 8 and 9 + = 12


= 2 and = 3


Hence the vertices of triangle are A(6 , 9), B(4 ,5) and C(2 ,3)



Question 23.

The line segment joining the points P (3, 3) and Q (6, - 6) is trisected at the points A and B such that A is nearer to P. If A also lies on the line given by 2x + y + k = 0, find the value of k.


Answer:

Here, given points are P (3, 3) and Q (6, - 6) which is trisected at the points(say) A(x1 , y1) and B(x2 , y2)such that A is nearer to P.



By section formula,


x = , y =


For point A(x1 , y1) of PQ, where m = 2 and n = 1,


x1 = , y1 =


∴ x1 = 4 , y1 = 0


∴Coordinates of A is (4,0)


It is given that point A lies on the line 2x + y + k = 0.


So, substituting value of x and y as coordinates of A,


2 × 4 + 0 + k = 0


∴ k = - 8



Question 24.

If the points (-2 , -1), (1, 0), (x, 3) and (1, y) form a parallelogram, find the values of x and y.


Answer:

Let given points be A(-2, -1), B(1, 0), C(x, 3), D(1, y) and let the intersection of diagonals be E(xm , ym )

It is given that □ABCD is a parallelogram.

By midpoint formula.

x = , y =

We know that midpoint of parallelogram coincide.

∴ Midpoint of AC = Midpoint of BD

∴ ( , ) = ( , )

= and =

∴x = 4 and y = 2


Question 25.

The points A (2, 0), B (9, 1), C (11, 6) and D (4, 4) are the vertices of a quadrilateral ABCD. Determine whether ABCD is a rhombus or not.


Answer:

Here given points are A (2, 0), B (9, 1), C (11, 6) and D (4, 4).


For a quadrilateral to be rhombus, all sides must be equal.



By distance formula,


XY =


For side AB,


AB =


=


= units.


For BC,


BC =


=


= units


CD =


=


= units.


AD =


=


= units.


Here all sides are unequal.


Hence □ABCD is not a rhombus.



Question 26.

If three consecutive vertices of a parallelogram are (1, -2), (3, 6) and (5, 10), find its fourth vertex.


Answer:

Let three vertices be A(1, -2), B(3, 6) and C(5, 10) and fourth vertex be D(x, y)


It is given that quadrilateral joining these four vertices is parallelogram, ie □ABCD is parallelogram.


We know that diagonals of parallelogram bisect each other, ie midpoint of the diagonals coincide.



Let E(xm , ym) be the midpoint of diagonals AC and BD.


By midpoint formula,


x = , y =


For diagonal AC,


xm = , ym =


∴ xm = , ym =


∴ E(xm , ym) ≡ (3, 4)


For diagonal BD,


3 = , 4 =


∴ x = 6 – 3 , y = 8 – 6


∴ x = 3 and y = 2


Hence, our fourth vertex is D(3 , 2)



Question 27.

If the points A(a, -11), B(5, b), C(2, 15) and D(1, 1) are the vertices of a parallelogram ABCD, find the values of a and b.


Answer:

Given: the points A(a, -11), B(5, b), C(2, 15) and D(1, 1) are the vertices of a parallelogram ABCD.

To find: the values of a and b.

Solution:


Given points are A(a, -11), B(5, b), C(2, 15) and D(1, 1) and let the intersection of diagonals be E(xm , ym )


It is given that □ABCD is a parallelogram.



By midpoint formula.


x = , y =


We know that midpoint of parallelogram coincide.


∴ Midpoint of AC = Midpoint of BD


∴ ( , ) = ( , )


= and =












⇒ a + 2 = 6 and 4 = b + 1

⇒ a = 6 - 2 and 4 - 1 = b

⇒ a = 4 and 3 = b

∴a = 4 and b = 3


Question 28.

If the coordinates of the mid-points of the sides of a triangle be (3, -2), (-3, 1) and (4, -3), then find the coordinates of its vertices.


Answer:

Let A(x1 , y1), B(x2 , y2) and C(x3 , y3) be the vertices of triangle.


Let D(3, -2), E(-3, 1) and F(4, -3) be the midpoints of sides BC, CA and AB respectively.



By midpoint formula.


x = , y =


For midpoint D(3, -2) of side BC,


3 = , -2 =


= 6 and = -4 …(1)


For midpoint E(-3, 1) of side CA,


-3 = , 1 =


= -6 and = 2 …(2)


For midpoint F(4, -3) of side AB,


4 = , -3 =


= 8 and =-6 …(3)


Adding 1,2 and 3, we get,


=6 -6 + 8


And = -4 + 2 -6


∴ 2(= 8 and 2() = -8


= 4 and = -4


+ 6 = 4 and – 4 = -4 …from 1


= -2 and = 0


Substituting above values in 3,


-2 + and 0 + = -6


= 10 and = -6


Similarly for equation 2,


-2 + = -6 and 0 + = 2


= -4 and = 2


Hence the vertices of triangle are A(-2 , 0), B(10 ,-6) and C(-4 ,2)



Question 29.

Find the lengths of the medians of a ABC having vertices at A (0,-1), B (2, 1) and C (0, 3).


Answer:

Here given vertices are A (0,-1), B (2, 1) and C (0, 3) and let midpoints of BC, CA and AB be D,E and F respectively.


By midpoint formula.



x = , y =


For midpoint D of side BC,


x = , y =


x = , y =


∴midpoint of side BC is D(1, 2)


For midpoint E of side AB,


x = , y =


x = , y =


∴midpoint of side AB is E(0, 1)


For midpoint F of side CA,


x = , y =


x = , y =


∴midpoint of side CA is F(1, 0)


By distance formula,


XY =


For median AD,


AD =


=


= units


For median BE,


BE =


=


= 2 units.


For median CF,


CF =


=


= units



Question 30.

Find the lengths of the medians of a ABC having vertices at A (5, 1), B (1, 5), and C(-3, -1).


Answer:

Here given vertices are A (0,-1), B (2, 1) and C (0, 3) and let midpoints of BC, CA and AB be D,E and F respectively.



By midpoint formula.


x = , y =


For midpoint D of side BC,


x = , y =


x = , y =


∴midpoint of side BC is D(-1, 2)


For midpoint E of side AB,


x = , y =


x = , y =


∴midpoint of side AB is E(1, 0)


For midpoint F of side CA,


x = , y =


x = , y =


∴midpoint of side CA is F(3, 3)


By distance formula,


XY =


For median AD,


AD =


=


= units


For median BE,


BE =


=


= 5 units.


For median CF,


CF =


=


= units



Question 31.

Find the coordinates of the points which divide the line segment joining the points (-4, 0) and (0, 6) in four equal parts.


Answer:

Let given coordinates be A(-4, 0) and B (0, 6).


We need to divide AB into 4 equal parts, ie first we need to find midpoint of AB, which will be D and then find out midpoints of AD and DB respectively.


Let required points be C(x1 , y1 ), D(xm , ym ) and E(x , y2 )



By midpoint formula.


x = , y =


For midpoint D of AB,


xm = , ym =


∴ xm = -2 and ym = 3


∴ D(xm , ym ) ≡ (-2 , 3)


Now, for midpoint C of AD,


X1 = , y2 =


x1 = -3 and y1 = 1.5


∴C(x1 , y1) ≡ (-3, 1.5)


For midpoint E of DB,


X2 = , y2 =


∴ x2 = -1 and y2 = 4.5


∴ D(x2 , y2 ) ≡ (-1 , 4.5)


Hence the co-ordinates of the points are (-3, 1.5), (-2 , 3) and (-1 , 4.5)



Question 32.

Show that the mid-point of the line segment joining the points (5, 7) and (3, 9) is also the mid-point of the line segment joining the points (8, 6) and (0, 10).


Answer:

Let given points be A(5, 7) and B(3, 9) and the points of other segment line be C(8, 6) and D(0, 10)



By midpoint formula.


x = , y =


For midpoint of AB,


x = , y =


x = 4 and y = 8 …(1)


Now for midpoint of CD,


e = , d = …(say)


∴ e = 4 and d = 8 …(2)


Here from 1 and 2 we say that midpoints of AB and CD are same, ie they coincide.



Question 33.

Find the distance of the point (1, 2) from the mid-point of the line segment joining the points (6, 8) and (2, 4).


Answer:

Let D(x, y) be the midpoints of A(6, 8) and B(2, 4). Let our third given point be C(1, 2).


By midpoint formula.


x = , y =


For midpoint D of AB,


x = and y =


∴ x =4 and y = 6


∴D(x, y) ≡ (4, 6)


Now to find distance between C and D,


By distance formula,


XY =


For CD,


CD =


=


= 5 units



Question 34.

If A and B are (1, 4) and (5, 2) respectively, find the coordinates of P when AP/BP = 3/4.


Answer:

Given points are A(1, 4) and B(5, 2). Let P be (x, y) and given ratio is 3:4.



By section formula,


x = , y =


For point P on AB,


x = , y =


x = and y =


Hence, required coordinates is P( , )



Question 35.

Show that the points A (1, 0), B (5, 3), C (2, 7) and D (-2, 4) are the vertices of a parallelogram.


Answer:

Let given points be A (1, 0), B (5, 3), C (2, 7) and D (-2, 4) and let the intersection of diagonals be E(xm , ym )



By midpoint formula.


x = , y =


For midpoint of diagonal AC,


X1 = , y1 =


∴x1 = , y1 =


∴ midpoint of diagonal AC is (x1, y1 ) ≡ (, ) …(1)


For midpoint of diagonal BD,


X2 = , y2 =


∴x2 = , y 2=


∴ midpoint of diagonal BD is (x2, y2 ) ≡ (, ) …(2)


Here, from 1 and 2 we say that midpoint of both the diagonals intersect at same point, ie (, )


But our intersection of diagonals is at E, which means that midpoint of diagonals intersect at single point, ie E(, )


We know that if midpoints of diagonals intersect at single point, then quadrilateral formed by joining the points is parallelogram.


Hence, our □ABCD is parallelogram.



Question 36.

Determine the ratio in which the point P (m, 6) divides the join of A(-4, 3) and B(2, 8). Also, find the value of m.


Answer:

Here, given points are A (-4, 3) and B ( 2, 8) and let the point dividing the line joining two points be P(m,6).


Let the ratio be m:n



By section formula,


x = , y =


For point P(m,6),


m = …(1)


And 6 = …(2)


Solving 2 for finding ratio between m and n,


6 =


6(m + n) = 8m +3n


6m + 6n = 8m +3n


∴ 2m = 3n


=


∴ m : n = 3 : 2


Now solving for equation 1, where m = 3 and n =2


m =


∴ m =


∴ m =


Hence, our point is (, 6)



Question 37.

Determine the ratio in which the point (-6, a) divides the join of A(-3, 1) and B(-8, 9). Also find the value of a.


Answer:

Here, given points are A(-3, 1) and B(-8, 9) and let the point dividing the line joining two points be C(-6,a).


Let the ratio be m:n



By section formula,


x = , y =


For point C(-6,a),


-6 = …(1)


And a = …(2)


Solving 1 for finding ratio between m and n,


-6 =


-6 (m + n) = -8m -3n


6m + 6n = 8m + 3n


∴ 2m = 3n


=


∴ m : n = 3 : 2


Now solving for equation 2, where m = 3 and n = 2


a =


a =


∴ a =


∴ a =


∴ value of a is



Question 38.

The line segment joining the points (3, -4) and (1, 2) is trisected at the points P and Q . If the coordinates of P and Q are (p, -2) and (5/3, q) respectively. Find the values of p and q.


Answer:

Let given points be A(3, -4) and B(1, 2) , which is trisected at points P(p, -2) and Q(5/3, q).

By section formula,

x = , y =
As point P divides the line in 1:2 and Q divides the line in 2:1.

For point P(p, -2)of AB, where m = 1 and n = 2,

p= ,
-2 =


Solving for p,

P =

For point Q(5/3, q) of AB, where m = 2 and n = 1,

= ,
q =

Solving for q,

q =

∴q = 0
Hence, the value of p and q are and 0 respectively.


Question 39.

The line joining the points (2,1) and (5,-8) is trisected at the points P and Q. If point P lies on the line 2x - y + k = 0. Find the value of k.


Answer:

Here, given points are P (2, 1) and Q (5, - 8) which is trisected at the points(say) A(x1 , y1) and B(x2 , y2)such that A is nearer to P.



By section formula,


x = , y =


For point A(x1 , y1) of PQ, where m = 1 and n = 2,


x1 = , y1 =


∴ x1 = 3 , y1 = -2


∴Coordinates of A is (3,-2)


It is given that point A lies on the line 2x - y + k = 0.


So, substituting value of x and y as coordinates of A,


2 × 3 – (-2) + k = 0


∴ k = -8



Question 40.

If A and B are two points having coordinates (-2, -2) and (2, -4) respectively, find the coordinates of P such that AP = AB.


Answer:

Given points are A(-2, -2) and B(2, -4). Let P be (x, y)


Here given that AP = AB.


But AB = AP + BP


∴7AP = 3AB


7AP = 3(AP + BP)


∴4AP = 3BP


=



By section formula,


x = , y =


For point P on AB, where m = 3 and n = 4


x = , y =


x = and y =


Hence, required coordinates is P( , )



Question 41.

Find the coordinates of the points which divide the line segment joining A (-2, 2) and B (2, 8) into four equal parts.


Answer:

Let given coordinates be A(-2, 2) and B (2, 8).


We need to divide AB into 4 equal parts, ie first we need to find midpoint of AB, which will be D and then find out midpoints of AD and DB respectively.


Let required points be C(x1 , y1 ), D(xm , ym ) and E(x , y2 )



By midpoint formula.


x = , y =


For midpoint D of AB,


xm = , ym =


∴ xm = 0 and ym = 5


∴ D(xm , ym ) ≡ (0 , 5)


Now, for midpoint C of AD,


X1 = , y2 =


x1 = -1 and y1 =


∴C(x1 , y1) ≡ (-1, )


For midpoint E of DB,


X2 = , y2 =


∴ x2 = 1 and y2 =


∴ E(x2 , y2 ) ≡ (1 , )


Hence the co-ordinates of the points are (-1, ) , (0 , 5) and (1 , )



Question 42.

A (4, 2), B (6, 5) and C (1, 4) are the vertices of ABC.

(i) The median from A meets BC in D. Find the coordinates of the point D.

(ii) Find the coordinates of point P on AD such that AP : PD = 2 :1.

(iii) Find the coordinates of the points Q and R on medians BE and CF respectively such that BQ : QE = 2 : 1 and CR : RF = 2 : 1.

(iv) What do you observe?


Answer:

(i) The median from A meets BC in D. Find the coordinates of the point D.


Here given vertices are A (4, 2), B (6, 5) and C (1, 4).



By midpoint formula.


x = , y =


For midpoint D of side BC,


x = , y =


x = , y =


Hence, the coordinates of D are ( , )


(ii) Find the coordinates of point P on AD such that AP : PD = 2 :1.



By section formula,


x = , y =


For point P on AD, where m = 2 and n = 1


x = , y =


∴ x = and y =


(iii) Find the coordinates of the points Q and R on medians BE and CF respectively such that BQ : QE = 2 : 1 and CR : RF = 2 : 1.


By midpoint formula.


x = , y =


For midpoint E of side AC,


x = , y =


x = , y =


Hence, the coordinates of E are ( , 3)


For midpoint F of side AB,


x = , y =


x = , y =


Hence, the coordinates of F are ( , )


By section formula,


x = , y =


For point Q on BE, where m = 2 and n = 1


x = , y =


∴ x = and y =


For point R on CF, where m = 2 and n = 1


x = , y =


∴ x = and y =


(iv) What do you observe?


We observe that the point P,Q and R coincides with the centroid.


This also shows that centroid divides the median in the ratio 2:1



Question 43.

ABCD is a rectangle formed by joining the points A (-1, -1), B (-1, 4), C (5, 4) and D (5,-1). P, Q, R and S are the mid-points of sides AB, BC, CD and DA respectively. Is the quadrilateral PQRS a square? a rectangle? or a rhombus? Justify your answer.


Answer:

Here given that A (-1, -1), B (-1, 4), C (5, 4) and D (5,-1).Also P, Q, R and S are the mid-points of sides AB, BC, CD and DA respectively.



By midpoint formula.


x = , y =


For midpoint P of side AB,


x = , y =


x = -1 , y =


Hence, the coordinates of P are (-1 , )


For midpoint Q of side BC,


x = , y =


x = 2 , y = 4


Hence, the coordinates of Q are (2 ,4)


For midpoint R of side CD,


x = , y =


x = 5 , y =


Hence, the coordinates of R are (5 , )


For midpoint S of side AD,


x = , y =


x = 2 , y = -1


Hence, the coordinates of S are (2 ,-1)


Now we find length of the length of the □PQRS,


By distance formula,


XY =


For PQ,


PQ =


=


= units


For QR,


QR =


=


= units


For RS,


RS =


=


= units


For PS,


PS =


=


= units


Here we can observe that all lengths of □PQRS are equal.


Now for diagonal PR,


PR =


=


= units


Now for diagonal QS,


QS =


=


= 5 units


Here in □PQRS, diagonals are unequal.


We know that a quadrilateral whose all sides are equal and diagonals are unequal, it is a rhombus.


Hence, our □PQRS is rhombus .



Question 44.

Show that A(-3, 2), B (-5, -5), C (2, -3) and D(4, 4) are the vertices of a rhombus.


Answer:

solution: Given points are A(-3, 2), B (-5, -5), C (2, -3) and D(4, 4)


Use distance formula


For AB ,


AB =






=


= units


For BC ,


BC =







=


= units


For CD,


CD =







=


= units


For AD ,


AD=







=


= units


Here we can observe that all lengths of □PQRS are equal.


Now for diagonal AC,


AC =







=


= units


Now for diagonal BD,


BD =







=


= units


AB = BC = CD = AD

And AC ≠ BD

Here in ABCD, diagonals are unequal.


We know that a quadrilateral whose all sides are equal and diagonals are unequal, it is a rhombus.


Hence, ABCD is rhombus .


Question 45.

Find the ratio in which the y-axis divides the line segment joining the points (5, -6) and (-1, -4). Also, find the coordinates of the point of division.


Answer:

Let our points be A(5, -6) and B(-1, -4).


Let point C(0, y) divide the line formed by joining by the points A and B in ratio of m:n.



By section formula,


x = , y =


For point C(0, y)


0 = , y =


Solving for x coordinate,


0 =


∴ m = 5n


=


∴ m : n = 5 : 1


Now solving for y coordinate, with m = 5 and n = 1,


y =


y =


∴ y =


There is no need to solve for x, as our point lies on y-axis


Hence, the coordinates of required point is C( , )



Question 46.

If the points A (6, 1), B (8, 2), C (9, 4) and D (k, p) are the vertices of a parallelogram taken in order, then find the values of k and p.


Answer:

Our given vertices are A(1, -2), B(3, 6) and C(5, 10) and fourth vertex be D(k, p)


It is given that quadrilateral joining these four vertices is parallelogram, ie □ABCD is parallelogram.


We know that diagonals of parallelogram bisect each other, ie midpoint of the diagonals coincide.


Let E(xm , ym) be the midpoint of diagonals AC and BD.



By midpoint formula,


x = , y =


For diagonal AC,


xm = , ym =


∴ xm = , ym =


∴ E(xm , ym) ≡ (, )


For diagonal BD,


= , =


∴ k = 15 – 8 , y = 5 – 2


∴ k = 7 and p = 3


Hence, our fourth vertex is D(7 , 3)



Question 47.

In what ratio does the point (-4, 6) divide the line segment joining the points A (-6, 10) and B(3, -8)?


Answer:

Given points are A (-6, 10) and B(3, -8)


Let the point C(-4, 6) divide AB in ratio m:n.



By section formula,


x = , y =


For point C(-4, 6) on the line joined by the points A and B.


-4 = …(1)


And, 6 = …(2)


Solving 1,


-4(m + n) = 3m - 6n


∴4m + 4n = -3m + 6n


∴ 7m = 2n


=


Hence, ratio is 2:7.



Question 48.

Find the coordinates of a point A, where AB is a diameter of the circle whose centre is (2, -3) and B is (1, 4).


Answer:

Here given that AB is a diameter of the circle whose centre is (say) C(2, -3) and B is (1, 4)


Let A be (x, y)


We know that as C is center, AC = CB or C is midpoint of AB.



By midpoint formula,


x = , y =


For Center C,


2 = and -3 =


∴ x = 4 – 1 and y = -6 – 4


∴ x = 3 and y = -10


Hence, coordinates of A are (3, -10)



Question 49.

A point P divides the line segment joining the points A (3, -5) and B (-4, 8) such that . If P lies on the line x + y = 0, then find the value of k.


Answer:

Here given points are A (3, -5) and B (-4, 8) .


Let point P be (x, y) which divides AB in ratio of k:1, also point P lies on line x + y = 0



By section formula,


x = , y =


For point P on the line joined by the points A and B.


x = , y =


Putting in given equation,


, = (x, y)

- 4 k + 3 + 8 k - 8 = 0

4k - 5 = 0

∴ 4k = 5


∴ k = 5/2





Question 50.

Find the ratio in which the point P(-1, y) line segment joining A ( -3,10) and B(6, -8) divides it. Also find the value of y.


Answer:

Here, given points are A ( -3,10) and B(6, -8) and the point dividing the line joining two points is P(-1,y).


Let the ratio be m:n



By section formula,


x = , y =


For point P(-1,a),


-1 = …(1)


And y = …(2)


Solving 1 for finding ratio between m and n,


-1 =


- (m + n) = 6m -3n


m + n = -6m + 3n


∴ 7m = 2n


=


∴ m : n = 2 : 7


Now solving for equation 2, where m = 2 and n = 7


y =


y =


∴ y =


∴ y =


∴ value of y is



Question 51.

Points p, Q, R and S divide the segment joining the points A (1, 2) and B (6, 7) in 5 equal parts. Find the coordinates of the points P, Q and R.


Answer:

Here given points are A (1, 2) and B (6, 7) which is divided into 5 equal parts by points P, Q, R and S


∴ AP = PQ= QR = RS = SB



The point P divides the line segment AB in the ratio 1:4.


By section formula,


x = , y =


For point P,


x = , y =


x = 2 and y = 3


∴ Coordinate of P is (2 ,3 )


The point Q divides the line segment AB in the ratio of 2:3.


For point Q,


x = , y =


x = 3 and y = 4


∴ Coordinate of Q is (3 ,4 )


The point R divides the line segment AB in the ratio of 3:2.


For point R,


x = , y =


x = 4 and y = 5


∴ Coordinate of R is (4 , 5 )



Question 52.

The mid-point P of the line segment joining the points A (- 10, 4) and B (- 2, 0) lies on the line segment joining the points C (- 9,-4) and D (- 4, y). Find the ratio in which P divides CD. Also, find the value of y.


Answer:

Here given points are A (- 10, 4) and B (- 2, 0)and the points of other segment line are C (- 9,-4) and D (- 4, y)


Let the point of intersection between AB and CD be P



By midpoint formula.


x = , y =


For midpoint of AB,


e = , d = …(say)


e = -6 and d = 2 …(1)


By section formula,


x = , y =


For point P on CD, where ratio is m:n,


-6 = and 2 =


Solving for m and n,


-6 =


∴-6(m + n) = -4m -9n


6m + 6n = 4m + 9n


2m = 3n


=


∴ Ratio is 3:2


Now solving for y, where m = 3 and n = 2,


2 =


∴ 3y -8 = 10


∴ 3y = 18


∴y = 6


Question 53.

Find the ratio in which the point P (x, 2) divides the line segment joining the points A (12,5) and B (4, -3). Also, find the value of x.


Answer:

Here, given points are A (12,5) and B (4, -3) and let the point dividing the line joining two points be P(x,2)


Let the ratio be m:n



By section formula,


x = , y =


For point P(x,2),


x = …(1)


And 2 = …(2)


Solving 2 for finding ratio between m and n,


2 =


2(m + n) = -3m +5n


2m + 2n = -3m +5n


∴ 5m = 3n


=


∴ m : n = 3 : 5


Now solving for equation 1, where m = 3 and n =5


x =


∴ x =


∴ x = 9


Hence, our point is (9, 2)



Question 54.

Find the ratio in which the line segment joining the points A (3,-3) and B (-2, 7) is divided by x-axis. Also, find the coordinates of the point of division.


Answer:

Our points are A (3,-3) and B (-2, 7)


Let point C(x, 0) divide the line formed by joining by the points A and B in ratio of m:n.



By section formula,


x = , y =


For point C(x, 0)


x = , 0 =


Solving for y coordinate,


0 =


∴ 7m -3n = 0


∴ 7m = 3n


=


∴ m : n = 3 : 7


Now solving for x coordinate, with m = 3 and n = 7,


x =


∴ x=


∴ x = =


Hence, the coordinates of required point is C(, 0)



Question 55.

Find the ratio in which the points P (3/4, 5/12) divides the line segments joining the points A(1/2 , 3/2) and B(2, -5).


Answer:

Given points are A(1/2 , 3/2) and B(2, -5)


Let the point P(3/4, 5/12) divide AB in ratio m:n.



By section formula,


x = , y =


For point P on the line joined by the points A and B.


= …(1)


And, = …(2)


Solving 1,


3(m + n) = 8m + 2n


∴3m + 3n = 8m + 2n


∴ 5m = n


=


Hence, ratio is 1:5.



Question 56.

If the points P, Q(x, 7), R, S(6, y) in this order divide the line segment joining A(2, p) and B (7, 10) in 5 equal parts, find x, y and p.


Answer:

Here given points are A (2, p) and B ( 7, 10) which is divided into 5 equal parts by points P, Q(x, 7), R and S(6, y)


∴ AP = PQ= QR = RS = SB



By section formula,


x = , y =


The point Q divides the line segment AB in the ratio of 2:3.


For point Q,


x = , 7 =


Solving above equations, we get,


x = 4 and p = 5


For point P, divides the line segment AB in the ratio 4:1.


6 = , y =


Solving for y and substituting value of p,


y =


∴ y = 9


Hence, values are x = 4, y = 9 and p = 5




Exercise 14.4
Question 1.

Find the centroid of the triangle whose vertices are:

(i) (1, 4), (-1, -1), (3, -2)

(ii) (- 2, 3), (2, -1), (4, 0)


Answer:

(i) (1, 4), (-1, -1), (3, -2)



We know that centroid of a triangle for the vertices (x1 , y1), (x2 , y2) and (x3 , y3) is


G(x, y) = ( , )


∴For coordinates (1, 4), (-1, -1), (3, -2),


Centroid of triangle = , )


= ( 1, )


Hence, centroid of triangle is ( 1, )


(ii) (- 2, 3), (2, -1), (4, 0)



We know that centroid of a triangle for (x1 , y1), (x2 , y2) and (x3 , y3) is


G(x, y) = ( , )


∴For coordinates (- 2, 3), (2, -1), (4, 0)


Centroid of triangle = , )


= ( , )


Hence, centroid of triangle is ( , )


Question 2.

Two vertices of a triangle are (1, 2), (3, 5) and its centroid is at the origin. Find the coordinates of the third vertex.


Answer:

Let the vertex of the triangle be A(1, 2), B(3, 5) and C(x, y)


Let the centroid be D(0, 0), as it is given that centroid is given at origin.


We know that centroid of a triangle for (x1 , y1), (x2 , y2) and (x3 , y3) is


C(x, y) = ( , )


For given coordinates A(1, 2), B(3, 5) and C(x, y), centroid is,


(0, 0) = , )


Solving for x and y,


1 + 3 +x =0 and = 0


∴ x = -4 and y = -7


Hence, the coordinate of third vertex is C(-4, -7)



Question 3.

Prove analytically that the line segment joining the middle points of two sides of a triangle is equal to half of the third side.


Answer:

Let ∆ABC be any triangle such that O is the origin.


∴Let coordinates be A(0, 0), B(x1 , y1), C(x2 , y2).


Let D and E are the mid-points of the sides AB and AC respectively.


We have to prove that line joining the mid-point of any two sides of a triangle is equal to half of the third side which means,


DE = BC


By midpoint formula,


x = , y =


For midpoint D on AB,


x =, y =


∴ x = and y =


∴ Coordinate of D is (, )


For midpoint E on AC,


x =, y =


∴ x = and y =


∴ Coordinate of E is ( , )


By distance formula,


XY =


For BC,


BC =


For DE,


DE =


= ( )


= BC


∴ DE = BC


Hence, we proved that line joining the mid-point of any two sides of a triangle is equal to half of the third side.



Question 4.

Prove that the lines joining the middle points of the opposite sides of a quadrilateral and the join of the middle points of its diagonals meet in a point and bisect one another.


Answer:

Let us consider a Cartesian plane having a parallelogram OABC in which O is the origin.


We have to prove that middle point of the opposite sides of a quadrilateral and the join of the mid-points of its diagonals meet in a point and bisect each other.


Let coordinates be A(0, 0).


So other coordinates will be B(x1 + x2, y1), C(x2, 0) ... refer figure.



Let P, Q, R and S be the mid-points of the sides AB, BC, CD, DA respectively.


By midpoint formula,


x = , y =


For midpoint P on AB,


x =, y =


∴ x = , y =


∴ Coordinate of P is ( , )


For midpoint Q on BC,


x =, y =


∴ x = , y =


∴ Coordinate of Q is ( , )


For R, we can observe that, R lies on x axis.


∴ Coordinate of R is ( , )


For midpoint S on OA,


x =, y =


∴ x = , y =


∴ Coordinate of S is ( , )


For midpoint of PR,


x = , y =


∴ x = , y =


∴ Midpoint of PR is ( , )


Similarly midpoint of QS is ( , )


Also, similarly midpoint of AC and OA is ( , )


Hence, midpoints of PR, QS, AC and OA coincide


∴We say that middle point of the opposite sides of a quadrilateral and the join of the mid-points of its diagonals meet in a point and bisect each other.



Question 5.

If G be the centroid of a triangle ABC and P be any other point in the plane, prove that PA2 + PB2 + PC2 = GA2 + GB2 + GC2 + 3 GP2.


Answer:

we will solve it by taking the coordinates A(x1,y1), B(x2,y2) and C(x3,y3)


Let the co ordinates of the centroid be G(u, v).


G(u, v) = ( , )


let the coordinates of P(h, k).


now we will find L.H.S and R.H.S. separately.


PA2+PB2+PC2


= (h - x1)2 +(k - y1)2 +(h - x2)2+ (k - y2)2 +(h - x3)2 +(k - y3)2 …by distance formula.


= 3(h2+k2)+(x12+x22+x32)+(y12+y22+y32)-2h(x1+x2+x3)-2k(y1+y2+y3)


= 3(h2+k2)+(x12+x22+x32)+(y12+y22+y32)-2h(3u)-2k(3v)


GA2+GB2+GC2+3GP2


= (u-x1)2+(v-y1)2+(u-x2)2+(v-y2)2+(u-x3)2+(v-y3)2+3[(u-h)2+(v-k)2] ……by distance formula.


= 3(u2+v2)+(x12+y12+x22+y22+x32+y32) - 2u(x1+x2+x3) - 2v(y1+y2+y3) + 3[u2+h2-2uh+v2+k2-2vk]


= 6(u2+v2)+(x12+y12+x22+y22+x32+y32)-2u(3u)-2v(3v) +3(h2+k2) -6uh-6vk


= (x12+x22+x32)+(y12+y22+y32)+3(h2+k2)-6uh-6vk


Hence LHS = RHS


(The above relation is known as Leibniz Relation)


Hence Proved.



Question 6.

If G be the centroid of a triangle ABC, prove that:

AB2 + BC2 + CA2 = 3 (GA2 + GB2 + GC2)


Answer:

We know that centroid of a triangle for (x1 , y1), (x2 , y2) and (x3 , y3) is


G(x, y) = ( , )


We assume centroid of ∆ABC at origin.


For x=0 and y=0


= 0 and = 0


= 0 and =0


Squaring on both sides, we get


x12 + x22 + x32 + 2x1x2 + 2x2x3 + 2x3x1 = 0 and y12 + y22 + y32 + 2y1y2 + 2y2y3 + 2y3y1 = 0 … (1)


AB2 + BC2 + CA2


= [(x2 – x1)2 + (y2 – y1)2] + [(x3 – x2)2 + (y3 – y2)2] + [(x1 – x3)2 + (y1 – y3)2]


= (x12 + x22 – 2x1x2 + y12 + y22 – 2y1y2)+(x22 + x32 – 2x2x3 + y22 + y32 – 2y2y3)+(x12 + x32 – 2x1x3 + y12 + y32 - 2y1y3)


= (2x12 + 2x22 + 2x32 – 2x1x2 – 2x2x3 – 2x1x3) + (2y12 + 2y22 + 2y32 – 2y1y2 – 2y2y3 – 2y1y3)


= (3x12 + 3x22 + 3x32) + (3y12 + 3y22 + 3y32) …from 1


= 3(x12 + x22 + x32) + 3(y12 + y22 + y32) … (2)


3(GA2 + GB2 + GC2)


= 3 [(x1 – 0)2 + (y1 – 0)2 + (x2 – 0)2 + (y2 – 0)2 + (x3 – 0)2 + (y3 – 0)2]


= 3 (x12 + y12 + x22 + y22 + x32 + y32)


= 3 (x12 + x22 + x32) + 3(y12 + y22 + y32) … (3)


From (2) and (3), we get


AB2 + BC2 + CA2 = 3(GA2 + GB2 + GC2)



Question 7.

If (-2, 3), (4, -3) and (4, 5) are the mid-points of the sides of a triangle, find the coordinates of its centroid.


Answer:

We know that centroid of ∆DEF will be the same that of ∆ABC as ∆DEF is formed by midpoints of ∆ABC.



∴ We know that centroid of a triangle for (x1 , y1), (x2 , y2) and (x3 , y3) is


G(x, y) = ( , )


∴ G(x, y) = ( , )


∴ G(x, y) = ( 2, )


Hence the centroid is ( 2, )



Question 8.

In Fig. 14.40, a right triangle BOA is given. C is the mid-point of the hypotenuse AB. Show that it is equidistant from the vertices 0, A and B.



Answer:

Given that ∆BOA is right angled triangle


By midpoint formula,


x = , y =


For midpoint C on AB,


x =, y =


∴ x = a and y = b


∴ Coordinates of C are (a, b)


It is given that C is the midpoint of AB.


By distance formula,


XY =


For OC,


OC =


= …(1)


For AC,


AC =


=


As C is midpoint, AC = CB. …(2)


Hence from 1 and 2, we say that is point C is equidistant from the vertices 0, A and B.



Question 9.

Find the third vertex of a triangle, if two of its vertices are at (-3, 1) and (0, -2) and the centroid is at the origin.


Answer:

Let the vertex of the triangle be A(1, 2), B(3, 5) and C(x, y)


Let the centroid be G(0, 0), as it is given that centroid is given at origin.



We know that centroid of a triangle for (x1 , y1), (x2 , y2) and (x3 , y3) is


G(x, y) = ( , )


For given coordinates A(1, 2), B(3, 5) and C(x, y), centroid is,


(0, 0) = , )


Solving for x and y,


-3 +x =0 and = 0


∴ x = 3 and y = 1


Hence, the coordinate of third vertex is C(3, 1).



Question 10.

A (3, 2) and B (-2, 1) are two vertices of a triangle ABC whose centroid G has the coordinates (5/3, - 1/3). Find the coordinates of the third vertex C of the triangle.


Answer:

Let the vertex of the triangle be A(3, 2), B(-2, 1) and C(x, y)


Let the centroid be G(, ), as it is given that centroid is given at origin.



We know that centroid of a triangle for (x1 , y1), (x2 , y2) and (x3 , y3) is


G(x, y) = ( , )


For given coordinates A(3, 2), B(-2, 1) and C(x, y)


(, ) = , )


Solving for x and y,


-3 + 2 +x =5 and 2 + = -1


∴ x = 6 and y = -4


Hence, the coordinate of third vertex is C(6, -4).



Exercise 14.5
Question 1.

Find the area of a triangle whose vertices are
(i) (6, 3), (-3, 5) and (4, - 2)
(ii)
(iii) (a, c + a), (a, c) and (-a, c - a)


Answer:

(i) (6, 3), (-3, 5) and (4, - 2)

Let A≡(x1, y1 ) ≡ (6, 3), B ≡ (x2, y2 ) ≡ (-3, 5) and C ≡ (x3, y3 ) ≡ (4, -2)

Area of ∆ABC = |x1 (y2 - y3) + x2 (y3 - y1) + x3 (y1 - y2)| sq. units

∴ Area of ∆ABC = | { 6 (5 – (-2)) – 3 ( -2 – 3) + 4 (3 – 5)} |

= | { 6 × 7 + 15 – 8 } |

= | 57 – 8|

= sq. units

(ii)

Area of the triangle having vertices (x1,y1), (x2,y2) and (x3,y3)

= |x1(y2-y3)+x2(y3-y1)+x3(y1-y2)|

Here, (x1, y1)=(at12,2at1), (x2,y2)=(at22,2at2), (x3,y3)=(at32,2at3)

∴, area=|at12(2at2-2at3)+at22(2at3-2at1)+at32(2at1-2at2)|

= |2a2t12t2-2a2t12t3+2a2t22t3-2a2t22t1+2a2t32t1-2a2t32t2|

= × 2a2|t12t2-t12t3+t22t3-t22t1+t32t1-t32t2|

=a2|t12t2-t12t3+t22t3-t22t1+t32t1-t32t2|

=a2|t12(t2-t3)+t2t3(t2-t3)-t1(t22-t32)|

=a2|t12(t2-t3)+t2t3(t2-t3)-t1(t2+t3)(t2-t3)|

=a2|(t2-t3)(t12+t2t3-t1t2-t1t3)|

=a2|(t2-t3){t1(t1-t2)-t3(t1-t2)}|

=a2|(t2-t3)(t1-t2)(t1-t3)|

∴ Area is a2|(t2-t3)(t1-t2)(t1-t3)| sq. units

(iii) (a, c + a), (a, c) and (-a, c - a)

Area of the triangle having vertices (x1,y1), (x2,y2) and (x3,y3)

= |x1(y2-y3)+x2(y3-y1)+x3(y1-y2)|

Area = | a (c – c + a) + a( c – a –c – a) – a (c + a – c)|

= | a (a ) + a(-2a) – a( a)|

= | -2a2 |

= a2

∴ Area is a2 sq. units


Question 2.

Find the area of the quadrilaterals, the coordinates of whose vertices are

(i) (-3, 2), (5, 4), (7, - 6) and (-5, - 4)

(ii) (1, 2), (6, 2), (5, 3) and (3, 4)

(iii) (-4, - 2), (-3, - 5), (3, - 2), (2, 3)


Answer:

(i) (-3, 2), (5, 4), (7, - 6) and (-5, - 4)


Let the vertices of the quadrilateral be A (−3, 2), B (5, 4), C (7, −6), and D (−5, −4). Join AC to form two triangles ∆ABC and ∆ACD.



Area of □ ABCD = Area of ∆ABC + Area of ∆ACD


Area of the triangle having vertices (x1,y1), (x2,y2) and (x3,y3)


= |x1(y2-y3)+x2(y3-y1)+x3(y1-y2)|


Area of ∆ABC


= |-3(4 – (-6))+5(-6 - 2)+7(2 - 4)|


= |-30 – 40 -14|


= 42 sq. units


Area of ∆ACD


= |-3(-6 – 4)+7(-4 - 2) - 5(2 + 6)|


= |6 – 42 - 40|


= 38 sq. units


Area of □ ABCD = 42 + 38 = 80 sq. units


(ii) (1, 2), (6, 2), (5, 3) and (3, 4)


Let the vertices of the quadrilateral be A (1, 2), B (6, 2), C (5, 3), and D (3, 4). Join AC to form two triangles ∆ABC and ∆ACD.



Area of □ ABCD = Area of ∆ABC + Area of ∆ACD


Area of the triangle having vertices (x1,y1), (x2,y2) and (x3,y3)


= |x1(y2-y3)+x2(y3-y1)+x3(y1-y2)|


Area of ∆ABC


= |1(2 – 3)+6(3 - 2)+5(2 - 2)|


= |-1 + 6|


= sq. units


Area of ∆ACD


= |1(3 – 4)+5(4 - 2) + 3(2 - 3)|


= |-1 + 10 -3|


= 3 sq. units


Area of □ ABCD = + 3 = sq. units


(iii) (-4, - 2), (-3, - 5), (3, - 2), (2, 3)


Let the vertices of the quadrilateral be A (−4, −2), B (−3, −5), C (3, −2), and D (2, 3). Join AC to form two triangles ∆ABC and ∆ACD



Area of □ ABCD = Area of ∆ABC + Area of ∆ACD


Area of the triangle having vertices (x1,y1), (x2,y2) and (x3,y3)


= |x1(y2-y3)+x2(y3-y1)+x3(y1-y2)|


Area of ∆ABC


= |-4(-5 – (-2)) - 3(-2 – (-2)) + 3(-2 – (-5))|


= |12 + 0 +9|


= sq. units


Area of ∆ACD


= |-4(-2 – 3) - 3(3 – (-2)) + 2(-2 – (-2))|


= |20 + 15 +0|


= sq. units


Area of □ ABCD = + = 28 sq. units



Question 3.

The four vertices of a quadrilateral are (1, 2), (-5, 6), (7, -4) and (k, -2) taken in order. If the area of the quadrilateral is zero, find the value of k.


Answer:

Let four vertices of quadrilateral be A (1, 2) and B (−5, 6) and C (7, −4) and D (k, −2)


Area of □ ABCD = Area of ∆ABC + Area of ∆ACD = 0 sq. unit


Area of the triangle having vertices (x1,y1), (x2,y2) and (x3,y3)


= |x1(y2-y3)+x2(y3-y1)+x3(y1-y2)|


Area of ∆ABC


= |1(6 – (-4)) - 5(-4 -2) + 7(2 – 6)|


= |10 + 30 -28|


= 6 sq. units


Area of ∆ACD


= |1(-2 – (-4)) + k(-4 -2) + 7(2 – (-2))|


= |2 - 6k + 30|


= (3k -15) sq. units


Area of ∆ABC + Area of ∆ACD = 0 sq. unit


∴ 6 + 3k -15 =0


3k -9 =0


∴ k =3


Hence, the value of k is 3



Question 4.

The vertices of ∆ ABC are (-2, 1), (5, 4) and (2, -3) respectively. Find the area of the triangle and the length of the altitude through A.


Answer:

Let three vertices be A (−2, 1) and B (5, 4) and C(2, −3)



Area of the triangle having vertices (x1,y1), (x2,y2) and (x3,y3)


= |x1(y2-y3)+x2(y3-y1)+x3(y1-y2)|


Area of ∆ABC


= |-2(4 – (-3)) + 5(-3 -1) + 2(1 – 4)|


= |-14 - 20 - 6|


= 20 sq. units


Now to find length of BC,


By distance formula,


XY =


For BC,


BC =


=


= sq. units


Area of ∆ABC = × Base × Altitude


∴ 20 = × × Altitude


∴ Altitude = units


Hence, the length of altitude through A is units.



Question 5.

Show that the following sets of points are collinear.

(a) (2, 5), (4, 6) and (8, 8)

(b) (1, -1), (2, 1) and (4, 5).


Answer:

(a) Let three given points be A(2, 5), B(4, 6) and C(8, 8).


Area of the triangle having vertices (x1,y1), (x2,y2) and (x3,y3)


= |x1(y2-y3)+x2(y3-y1)+x3(y1-y2)|


Area of ∆ABC


= |2(6 – 8) + 4(8 -5) + 8(5 – 6)|


= |-4 + 12 - 8|


= 0 sq. units


We know that if area enclosed by three points is zero, then points are collinear.


Hence, given three points are collinear.


(b) Let three given points be A(1, −1), B(2, 1) and C(4, 5)


Area of the triangle having vertices (x1,y1), (x2,y2) and (x3,y3)


= |x1(y2-y3)+x2(y3-y1)+x3(y1-y2)|


Area of ∆ABC


= |1(1 – 5) + 2(5 + 1) + 4(-1 – 1)|


= |-4 + 12 - 8|


= 0 sq. units


We know that if area enclosed by three points is zero, then points are collinear.


Hence, given three points are collinear.



Question 6.

Prove that the points (a, 0), (0, b) and (1, 1) are collinear if,


Answer:

Let three given points be A(a,0), B(0,b) and C(1,1).


Area of the triangle having vertices (x1,y1), (x2,y2) and (x3,y3)


= |x1(y2-y3)+x2(y3-y1)+x3(y1-y2)|


Area of ∆ABC


= |a(b – 1) + 1(0 -b)|


= | ab – a –b|


Here given that


= 1


∴ a + b = ab


Now,


Area of ∆ABC


= | ab - (a + b)|


= | ab – ab|


= | 0 |


= 0 sq. units


We know that if area enclosed by three points is zero, then points are collinear.


Hence, given three points are collinear.



Question 7.

The point A divides the join of P (-5, 1) and Q (3, 5) in the ratio k : 1. Find the two values of k for which the area of a ABC where B is (1, 5) and C (7, -2) is equal to 2 units.


Answer:

coordinates A can be given by using section formula for internal division,


A = ( , )


and B (1,5), C (7,−2)


Area of the triangle having vertices (x1,y1), (x2,y2) and (x3,y3)


= |x1(y2-y3)+x2(y3-y1)+x3(y1-y2)|


Area of ∆ABC


= | (7) + 1(−2 − ) + 7( - 5 ) |


But Area of ∆ABC = 2


| (7) + 1(−2 − ) + 7( - 5 ) | = 2


Solving above we get,


| | = 4


Taking positive sign, 14k−66=4k+4


10k = 70


k=7


Taking negative sign we get,


14k−66=−4k−4


18k = 62


k = =



Question 8.

The area of a triangle is 5. Two of its vertices are (2, 1) and (3, -2). The third vertex lies on y = x + 3. Find the third vertex.


Answer:

Let ABC be a triangle with A(a, b),B(2,1) and C(3,-2).


A lies on the line y=x+3 means,


b=a+3 ...(1).


Area of the triangle having vertices (x1,y1), (x2,y2) and (x3,y3)


= |x1(y2-y3)+x2(y3-y1)+x3(y1-y2)|


Area of ∆ABC = 5


Substituting the values of A, B and C in formula, we, get,


5 = | 3a + b – 7 |


Taking positive value for | 3a + b – 7 |,


3a+b=17 ….(2)


Solving 1 and 2 simultaneously,


a = and b =


Hence coordinates of the vertex A are ( , ).


Taking negative value for | 3a + b – 7 |,


(3a+b−7) = − 5


3a+b=−3 …(3)


Solving 1 and 2 simultaneously,


A = and b = and the vertex A is (,)


Hence the coordinates of third vertex are ( , ) or (,).



Question 9.

If , prove that the points (a, a2), (b,b2),(c, c2) can never be collinear.


Answer:

Area of the triangle having vertices (x1, y1), (x2, y2) and (x3, y3) is given by



For points to be collinear, the Area enclosed by them should be equal to 0

∴ For given points,


Area = 1/2 |(b – c)(a - b)(c – a)|

Area ≠ 0

Also it is given that .

Hence area of triangle made by these points is never zero. Hence given points are never collinear.


Question 10.

Four points A (6, 3), B (-3, 5), C (4, - 2) and D (x, 3x) are given in such a way that , find x.


Answer:

Four points A (6, 3), B (−3, 5) C (4, −2) and D(x, 3x)

Area of the triangle having vertices (x1,y1), (x2,y2) and (x3,y3)

= |x1(y2-y3)+x2(y3-y1)+x3(y1-y2)|

Area of ∆ABC

= |6(5 – (- 2)) - 3(-2 -3) + 4(3 – 5)|

= |42 + 15 -8|

= sq. units

Area of ∆DBC

= |x(5 – (-2)) + 3(-2 -3x) + 4(3x - 5))|

= |7x +6 + 9x + 12x - 20|

= | 28x -14|

= ± 7(2x -1 )

It is given that

∴2 × ∆DBC = ∆ABC

2 × (± 7(2x -1 )) =

∴ ± 4(2x -1 ) = 7

∴ 4(2x -1 ) = 7 or -4(2x -1 ) = 7

∴ 8x – 4 =7 or -8x + 4 =7

∴ 8x =11 or -8x =3

∴x= or x =

Hence, the value of x is or


Question 11.

For what value of a the point (a, 1),(1, -1) and(11, 4) are collinear?


Answer:

The three given points are A(a, 1), B(1, −1) and C(11, 4).


Area of the triangle having vertices (x1,y1), (x2,y2) and (x3,y3)


= |x1(y2-y3)+x2(y3-y1)+x3(y1-y2)|


Given that area of ∆ABC = 0


∴ 0 = |a(-1 – 4) + 1(4 - 1) + 11(1 – (-1))|


∴ 0 = |-5a + 3 + 22|


∴ -5a + 3 + 22 = 0


a = 5


Hence the value of a is 5



Question 12.

Prove that the points (a, b),(a1,b1) and (a - a1, b - b1) are collinear if ab1 = a1b


Answer:

Consider the following points A(a,b), B(a1,b1), C(a−a1,b−b1)


Since the given points are collinear, we have area(△ABC)=0


First find the area of area(△ABC) as follows:


area(△ABC)=1/2 |x1(y1−y3)+x1(y3−y1)+x3(y1−y1)|


=|a(b1−(b−b1))+a1((b−b1)−b)+(a−a1)(b−b1)|


= |a(b1−b+b1)+a1(b−b1−b)+a(b−b1)−a1(b−b1)|


=|−ab−a1b1+ab−ab1+a1b+a1b1|


=|−(ab1−a1b)|


= (ab1−a1b)


This gives, ab1−a1b=0


∴ ab1 = a1b



Question 13.

If three points (x1, y1), (x2, y2), (x3, y3) lie on the same line, prove that


Answer:

Area of the triangle having vertices (x1,y1), (x2,y2) and (x3,y3)


= |x1(y2-y3)+x2(y3-y1)+x3(y1-y2)|


Given that all points are collinear.


∴ area = 0


x1(y2-y3)+x2(y3-y1)+x3(y1-y2) = 0


Dividing by x1 x2 x3,


+ + = 0



Hence proved.



Question 14.

If (x, y) be on the line joining the two points (1, -3) and (-4, 2), prove that x+y+2=0.


Answer:

Given: The point (x, y) is on the line joining the two points (1, -3) and (-4, 2).
To Prove: x+y+2=0
Proof: When the points line on the same line they are called collinear points.
As the point (x, y) lies on the line joining the points (1, −3) and (−4, 2), it means that the three points are collinear.
If the points are in same straight line they cannot form a triangle which implies that area of triangle becomes zero.
If the vertices of the triangle are given in the form of (a,b)
where a and b are the coordinates of a given point in the direction of x and y axis respectively.

Area of the triangle having vertices (x1,y1), (x2,y2) and (x3,y3) is given as:

.......
(1)

Now, for the three points to be collinear,


Now if the points (x, y), (1, −3) and (−4, 2) are collinear,the area of the triangle formed by these points is zero.
Substitute the given values in equation (1),
So,

-5x + 2 –y -4y -12 = 0

-5x -5y -10 = 0

Taking "-5" common from the equation we get,

⇒ -5(x+y+2)=0

⇒ (x+y+2)=0

Hence proved, (x+y+2)=0
Conclusion: If (x, y) be on the line joining the two points (1, -3) and (-4, 2), then x+y+2=0.


Question 15.

Find the value of k if points (k, 3), (6, - 2) and (-3, 4) are collinear.


Answer:

The three given points are A(k, 3), B(6, −2) and C(3, 4). It is also said that they are collinear and hence the area enclosed by them should be 0.


Area of the triangle having vertices (x1,y1), (x2,y2) and (x3,y3)


= |x1(y2-y3)+x2(y3-y1)+x3(y1-y2)|


Given that area of ∆ABC = 0


∴ 0 = |k(-2 – 4) + 6(4 - 3) - 3(3 – (-2))|


∴ 0 = |-6k + 6 - 15|


∴ - |-6k + 9|= 0


6k + 9 = 0


∴ k =


Hence, the value of k is



Question 16.

Find the value of k, if the points A (7, -2), B (5, 1) and C (3, 2k) are collinear.


Answer:

The three given points are A(7, −2), B(5, 1) and C(3, 2k). It is also said that they are collinear and hence the area enclosed by them should be 0.


Area of the triangle having vertices (x1,y1), (x2,y2) and (x3,y3)


= |x1(y2-y3)+x2(y3-y1)+x3(y1-y2)|


Given that area of ∆ABC = 0


∴ 0 = |7(1 – 2k) + 5(2k – (-2)) + 3(-2 – 1)|


∴ 0 = |7 – 14k + 10k +10 -6 -3|


∴ - |8 – 4k|= 0


8 – 4k = 0


-4k = -8


∴ k = 2



Question 17.

If the point P (m, 3) lies on the line segment joining the points A( and B (2, 8), find the value of m.


Answer:

It is said that the point P(m,3) lies on the line segment joining the points A( and B (2, 8).

Hence we understand that these three points are collinear. So the area enclosed by them should be 0.

Area of the triangle having vertices (x1,y1), (x2,y2) and (x3,y3)

= |x1(y2-y3)+x2(y3-y1)+x3(y1-y2)|

Given that area of ∆ABP = 0

∴ 0 = |m(6 – 8) - (8 – 3) + 2(3 – 6)|

∴ -2m – 2 -6 = 0

-2m = 8

m = -4

Hence the value of m = -4


Question 18.

If R (x, y) is a point on the line segment joining the points P (a, b) and Q (b, a), then prove that x + y = a + b.


Answer:

Given : R (x, y) is a point on the line segment joining the points P (a, b) and Q (b, a).
To prove: x + y = a + b
Proof:

It is said that the point R(x, y) lies on the line segment joining the points P(a, b) and Q(b, a). Thus, these three points are collinear.

So the area enclosed by them should be 0.

Area of the triangle having vertices (x1,y1), (x2,y2) and (x3,y3) is:

Given that area of ∆PQR = 0

|x(b – a) + a(a – y) + b(y – b)| = 0

∴ bx – ax + a2 - ay + by - b2 = 0

∴ ax + ay –bx – by - a2 - b2 = 0

∴ ax + ay –bx – by = a2 + b2

(a – b)(x + y) = (a – b )(a + b)

∴ x +y = a + b

Hence proved.


Question 19.

Find the value of k, if the points A (8, 1), B (3, - 4) and C (2, k) are collinear.


Answer:

Given points are A(8,1),B(3,−4) and C(2,k).It is also said that they are collinear and hence the area enclosed by them should be 0.


Area of the triangle having vertices (x1,y1), (x2,y2) and (x3,y3)


= |x1(y2-y3)+x2(y3-y1)+x3(y1-y2)|


Given that area of ∆ABC = 0


∴ 0 = |8(-4 – k) + 3(k – 1) + 2(1 – (-4))|


∴ 0 = |-32 – 8k + 3k -3 + 10|


∴ 5k + 25 = 0


∴ k = -5


Hence, the value of k is -5.



Question 20.

Find the value of a for which the area of the triangle formed by the points A (a, 2a), B (-2, 6) and C (3, 1) is 10 square units.


Answer:

Given points are A(a,2a), B(2,6) and C(3,1). It is also said that the area enclosed by them is 10 square units.

Area of the triangle having vertices (x1,y1), (x2,y2) and (x3,y3)

= |x1(y2-y3)+x2(y3-y1)+x3(y1-y2)|

Given that area of ∆ABC = 10

∴ 10 = |a(6 – 1) -2 (1 – 2a) + 3(2a - 6)|

∴ 20 = |5a – 2 + 4a + 6a - 18|

∴ 20 = | 15a – 20|

∴ 15a – 20 = ± 20

Taking positive sign,

15a – 20 = 20

a =

Taking negative sign,

15a – 20 = -20

a = 0

Hence, the value of a are 0 and


Question 21.

If the vertices of a triangle are (1,-3), (4, p) and (-9, 7) and its area is 15 sq. units, find the value(s) of p.


Answer:

Let A(1, −3), B(4, p) and C(−9, 7) be the vertices of the ∆ABC.

Area of the triangle having vertices (x1,y1), (x2,y2) and (x3,y3)

= |x1(y2-y3)+x2(y3-y1)+x3(y1-y2)|

Given that area of ∆ABC = 15

∴ 15 = |1(p – 7) +4 (7 – (-3)) - 9(-3 - p)|

∴ 30 = |p – 7 + 40 + 27 + 9p|

∴ 30 = | 10p + 60|

∴ 10p + 60 = ± 30

Taking positive sign,

10p + 60 = 30

p = -3

Taking negative sign,

10p + 60 = - 30

p = -9

Hence, the value of p are -3 and -9


Question 22.

Find the area of a parallelogram ABCD if three of its vertices are A(2, 4), B (2 + , 5) and C(2, 6).


Answer:

It is given that A(2, 4), B(2 + , 5) and C(2, 6) are the vertices of the parallelogram ABCD.


Area of the triangle having vertices (x1,y1), (x2,y2) and (x3,y3)


= |x1(y2-y3)+x2(y3-y1)+x3(y1-y2)|


Area of □ABCD = 2 × Area of ∆ABC


Area of ∆ABC = |2(5 - 6)+ (2 + )(6 - 4)+2(4 - 5)|


= |-2 + 4 + - 2|


= × = sq. units


∴ Area of □ABCD = 2 × = sq. units


Hence, the area of given parallelogram is sq. units



Question 23.

Find the value (s) of k for which the points (3k - 1, k - 2), (k, k - 7) and (k - 1,-k - 2) are collinear.


Answer:

Let A ( 3k − 1, k − 2 ) , B ( k, k − 7 ) and C ( k − 1, −k − 2 ) be the given points.
For points to be collinear area of triangle formed by the vertices must be zero.

Area of the triangle having vertices ( x1,y1 ) , ( x2,y2 ) and ( x3,y3 ) = |x1 ( y2 - y3 ) + x2 ( y3 - y1 ) + x3 ( y1 - y2 ) |

area of ∆ABC = 0

⇒ ( 3k−1 ) [ ( k−7 ) − ( −k−2 ) ] + k [ ( −k−2 ) − ( k−2 ) ] + ( k−1 ) [ ( k−2 ) − ( k−7 ) ] =0
⇒ ( 3k−1 ) [ k−7 + k + 2 ] + k [ −k−2 − k+ 2 ] + ( k−1 ) [ k−2 − k + 7 ] =0

⇒ ( 3k−1 ) ( 2k−5 ) + k (−2k ) + 5 ( k −1 ) =0
⇒ 6k2 - 15k -2k + 5 - 2k2 + 5k - 5 = 0

⇒ 6k2−17k + 5−2k2 + 5k−5=0

⇒ 4k2−12k=0

⇒ 4k ( k−3 ) =0

⇒ k=0 or k−3=0

⇒ k=0 or k=3


Hence, the value of k is 0 or 3.


Question 24.

If the points A (-1,-4), B (b,c) and C (5,-1) are collinear and 2b + c = 4, find the values of b and c.


Answer:

The given points A(−1, −4), B(b, c) and C(5, −1) are collinear.


Area of the triangle having vertices (x1,y1), (x2,y2) and (x3,y3)


= |x1(y2-y3)+x2(y3-y1)+x3(y1-y2)|


Given that area of ∆ABC = 0


∴ −1[c − (− 1)]+b[− 1 − ( − 4)] + 5( − 4 − c) = 0


∴ − c − 1 + 3b − 20 − 5c = 0


3b − 6c = 21


∴b − 2c = 7 …(1)


Also it is given that 2b + c = 4 …(2)


Solving 1 and 2 simultaneously, we get,


2(7 + 2c) + c = 4


14 + 4c + c = 4


5c = − 10


c = − 2


∴ b = 3


Hence, value of b and c are 3 and -2 respectively



Question 25.

If the points A (-2,1), B (a, b) and C (4,-1) are collinear and a – b = 1, find the values of a and b.


Answer:

The given points A(−2, 1), B(a, b) and C(4, −1) are collinear.


Area of the triangle having vertices (x1,y1), (x2,y2) and (x3,y3)


= |x1(y2-y3)+x2(y3-y1)+x3(y1-y2)|


Given that area of ∆ABC = 0


∴ −2[b − ( − 1)] + a( − 1 – 1 ) + 4 ( 1 –b ) = 0


-2b – 2 − 2a + 4 − 4b = 0


− 2a − 6b = − 2


a + 3b = 1 …(1)


Also it is given that a – b = 1 …(2)


Solving 1 and 2 simultaneously,


B + 1 + 3b = 1


4b = 0


∴ b = 0


∴ a =1


Hence, the values of a and b are 1 and 0.



Question 26.

If A (-3, 5), B (-2,-7), C (1,-8) and D (6, 3) are the vertices of a quadrilateral ABCD, find its area.


Answer:

Given vertices of a quadrilateral ABCD are A(−3, 5), B(−2, −7), C(1, −8) and D(6, 3)


Area of the quadrilateral ABCD = Area of ∆ABC + Area of ∆ACD


Area of the triangle having vertices (x1,y1), (x2,y2) and (x3,y3)


= |x1(y2-y3)+x2(y3-y1)+x3(y1-y2)|


Area of ∆ABC = | − 3[ − 7 − ( − 8 )] + ( − 2) ( − 8 – 5 ) + 1 [ 5 − (− 7) ] |


= | − 3 + 26 + 12 |


=sq. units


Area of ∆ACD = |− 3( − 8 – 3 ) + 1( 3 – 5 ) + 6[ 5 − ( − 8 ) ] |


= | 33 – 2 + 78 |


= sq. units


Area of the quadrilateral ABCD = + = 72 sq. units


∴Hence, the area of the quadrilateral is 72 sq. units.



Question 27.

If P (-5, - 3), Q (-4, -6), R (2,-3) and S (1, 2) are the vertices of a quadrilateral PQRS, find its area.


Answer:

Let P(−5,−3); Q(−4,−6); R(2,−3) and S(1,2) be the vertices of quadrilateral PQRS.


Area of the quadrilateral PQRS = Area of ∆PQR + Area of ∆PSR


Area of the triangle having vertices (x1,y1), (x2,y2) and (x3,y3)


= |x1(y2-y3)+x2(y3-y1)+x3(y1-y2)|


Area of ∆PQR = |− 5( − 6 + 3 ) − 4( − 3 + 3 ) + 2( −3 + 6) |


= |15 + 0 + 6|


= sq. units


Area of ∆PSR = | − 5( 2 + 3 ) + 1( − 3 + 3 ) + 2( − 3 − 2) |


= | − 25 + 0 – 10 |


= sq. units


Area of the quadrilateral PQRS = + = 28 sq. units


∴Hence, the area of the quadrilateral is 28 sq. units.


(given answer is wrong, its not 13, it is 28 )



Question 28.

Find the area of the triangle PQR with Q (3, 2) and the mid-points of the sides through Q being (2, -1) and (1, 2).


Answer:

Let the co-ordinates of P and R be (a,b) and (c,d) and coordinates of Q are (3, 2)


By midpoint formula.


x = , y =


(2 , - 1) is the mid-point of PQ.


∴ 2 = and -1 =


∴ a = 1 and b = -4


∴ Coordinates of P are (1, -4)


(1 , 2) is the mid-point of QR.


∴ 1 = and 2 =


∴ c = -1 and d = 2


∴ Coordinates of P are (-1, 2)


Area of the triangle having vertices (x1,y1), (x2,y2) and (x3,y3)


= |x1(y2-y3)+x2(y3-y1)+x3(y1-y2)|


Area of ∆PQR = | 3( − 4 – 2 ) + 2( − 1 – 1) + 1( 2 − 4) |


= | − 18 − 4 − 2 |


= 12 sq. units


Hence the area of ∆PQR is 12 sq. units